Cosmetic Rhinoplasty 01-22 Flashcards

1
Q

A patient presents for consultation for reconstruction of a saddle deformity following nasal trauma. Structural support for dorsal augmentation and lengthening of the nose is required. Which of the following is most likely to result in secondary development of nasal asymmetry during the early postoperative period?

A) Alloplastic implant
B) Cantilevered costal bone graft
C) Cantilevered cranial bone graft
D) Carved costal cartilage graft
E) Diced cartilage graft

A

The correct response is Option D.

Multiple techniques have been described for dorsal augmentation in rhinoplasty. Carved costal cartilage offers structural support for both dorsal augmentation and lengthening of the nose. There is a large amount of donor material available, but carved rib cartilage is prone to warping and may result in the development of secondary deformity. Techniques, such as concentric carving and insertion of a k-wire into the graft, have been used to mitigate this, but success is very technique-dependent. Dorsal augmentation with bone grafts, such as cranial bone graft and rib, are more subject to bone resorption than warping. Diced cartilage grafts offer dorsal augmentation but do not offer structural support to lengthen the nose. These grafts are malleable in the postoperative period and potentially allow correction of minor asymmetries.

Nasal augmentation with alloplastic implants is common in Asian countries. These implants carry a risk for infection, extrusion, capsule contracture, and malposition. While rare, these complications can be severe and result in secondary deformity.

How well did you know this?
1
Not at all
2
3
4
5
Perfectly
2
Q

A 28-year-old woman undergoes a septorhinoplasty. Fracture of the septal L-strut will most likely result in which of the following complications?

A) Bossa formation
B) Open roof deformity
C) Pollybeak deformity
D) Rocker deformity
E) Saddle-nose deformity

A

The correct response is Option E.

The keystone area of the nose is the junction of the cartilaginous and bony dorsum where the paired upper lateral cartilages, septal cartilage, bony septum, and paired nasal bones meet. Much like the keystone in an archway, if this area is damaged, the archway collapses. In the case of the nose, a fracture of the septal cartilage and the bony cartilage can result in instability and collapse of the septal cartilage and middle third of the nose into the pyriform, resulting in the saddle-nose deformity. When performing a septoplasty, preservation of a 1- cm caudal × 1-cm dorsal L-strut is recommended to preserve structural integrity. Regardless of the size of the L-strut, an unrecognized intraoperative fracture can result in a saddle-nose deformity.

Bossa are knob-like projections of alar cartilage irregularities through the skin. These can form as a result of overaggressive suturing, resulting in buckling or flexing of the cartilage, cutting of the cartilages, or scar tissue contraction of the cartilages.

An open roof deformity can be seen after a dorsal hump reduction. This is the result of seeing the separation between the sidewalls of the nose and the septum through the nasal skin as the skin falls unsupported into the grooves. This deformity can be corrected either by narrowing the separation by infracture of the nasal bones or by filling the defect with spreader grafts.

A pollybeak deformity is fullness in the supratip region of the nose. This can appear as a result of ptosis of the tip in relationship to distal dorsal septum, resection of the distal dorsal septum, or scar formation in the supratip region.

A rocker deformity can occur after nasal osteotomies that go into the thicker bones above the level of the canthus and into the nasofrontal region. When the distal nasal bone is infractured medially, the superior portion of the segment moves (rocks out) laterally.

How well did you know this?
1
Not at all
2
3
4
5
Perfectly
3
Q

A 39-year-old man undergoes rhinoplasty at the practice. Six months after the procedure, he is displeased with the outcome and notes mild persistent tip deviation. After seeking three opinions from plastic surgeons in the community who all agree to care for him, he returns to the practice and insists that the original surgeon fix his nose because he knows that surgeon is “the best.” This patient most likely has which of the following psychological disorders?

A) Bipolar disorder
B) Body dysmorphic disorder
C) Histrionic personality disorder
D) Narcissistic personality disorder

A

The correct response is Option D.

Identification of various personality or psychiatric disorders prior to administering aesthetic care for patients can minimize social issues that can arise with the physician-patient relationship. Body dysmorphic disorder is hallmarked by pervasive and intrusive thoughts about a physical element (arm contour, for example) and may occupy several hours a day, causing severe distress and impairing one’s otherwise normal activities. The diagnosed individual is often the only person who can see the affliction. Histrionic personality disorder (HPD) is characterized by a pattern of excessive attention-seeking behaviors. This includes inappropriate seduction and an excessive desire for approval. People diagnosed with HPD are often lively, dramatic, vivacious, enthusiastic, and flirtatious. Narcissistic personality disorder (NPD) patients have a grandiose sense of self-importance and are preoccupied with fantasies of unlimited success, power, brilliance, beauty, or ideal love. These patients have a sense of entitlement (i.e., unreasonable expectations of especially favorable treatment or automatic compliance with their expectations). The ironic hallmark of the patient with NPD is that they often return for cosmetic surgery even if they are dissatisfied with the initial results. Bipolar disorder patients have extremes in mood/behavior: mania and depression. This mental illness is hallmarked by severe high and low moods and changes in sleep, energy, thinking, and behavior. These patients have periods of feeling overly happy and energized and other periods of feeling very sad, hopeless, and sluggish.

How well did you know this?
1
Not at all
2
3
4
5
Perfectly
4
Q

A 32-year-old man wants both cosmetic and functional improvement of his nose. He is scheduled for open rhinoplasty, submucous resection of the septum, spreader grafts, and submucous reduction of the inferior turbinates. Which of the following can be mixed with the local anesthesia to best decrease intraoperative bleeding during the rhinoplasty?

A) Aprotinin
B) Hyaluronic acid
C) Ketorolac
D) Tissue plasminogen activator
E) Tranexamic acid

A

The correct response is Option E.

Tranexamic acid inhibits fibrinolysis, which strengthens clots and decreases bleeding. Tranexamic acid has been shown to decrease bleeding in many plastic surgical procedures including rhinoplasty, rhytidectomy, liposuction, reduction mammaplasty, and abdominoplasty, as well as others. The method of giving tranexamic acid includes intravenous, oral, topical, and local infiltration. Studies have failed to show an increased risk for thromboembolic events. The therapeutic results last for at least 17 hours. When mixed with local anesthesia, the most common doses are 1 to 2 mg/mL. Use of tranexamic acid in plastic surgery is increasingly popular but is presently off-label.

Aprotinin also prohibits fibrinolysis; however, this drug was removed from the market in the United States because of increased mortality associated with its use. Tissue plasminogen activator activates the conversion of plasminogen to plasmin, which dissolves clots and can increase bleeding. Ketorolac is a nonsteroidal anti-inflammatory drug and inhibits platelet function. Hyaluronic acid has not been shown to decrease bleeding.

How well did you know this?
1
Not at all
2
3
4
5
Perfectly
5
Q

A 62-year-old married man presents for rhinoplasty with concerns for airway obstruction. He reports that he is generally unhappy with his nasal tip and that the appearance interferes with his work relationships. Which of the following patient characteristics is the strongest indicator for body dysmorphic disorder?

A) Age
B) Concerns of airway obstruction
C) Marital status
D) Report that nose affects his work relationships
E) Unhappiness with nasal tip

A

The correct response is Option D.

Surgeons should avoid patients who are described as “SIMON,” single, immature, male, overly expectant, and narcissistic. Male patients can have more difficulty describing their specific concerns with the aesthetic appearance of their noses, and it can be very challenging to meet expectations. They may also disguise their true motives involving rhinoplasty.

The correct answer is the patient’s report that his nose affects his work relationships. People with body dysmorphic disorder (BDD) will place a very large emphasis on the physical area that they are focused on and believe that changing the area will improve their relationships.

Concerns of airway obstruction are incorrect because male patients may disguise their true motives for seeking a rhinoplasty; this is a valid concern and can be investigated during the clinical examination.

This patient is married, which is not known to be risk factor for male rhinoplasty patients and BDD. v His age is not a known risk factor for male rhinoplasty patients and BDD.

Having a complaint about a specific part of the nose is not a known risk factor for BDD

How well did you know this?
1
Not at all
2
3
4
5
Perfectly
6
Q

A 45-year-old man presents 1 year after a “nose surgery” for a crooked nose and reports subjective nasal obstruction. He is diagnosed with empty nose syndrome. Overresection of which of the following structures is the most likely cause of this patient’s condition?

A) Inferior turbinates
B) Lower lateral cartilage
C) Middle turbinates
D) Septal cartilage
E) Upper lateral cartilage

A

The correct response is Option A.

Empty nose syndrome is characterized by a subjective sensation of nasal obstruction in the context of adequate nasal space. The differential diagnosis includes chronic rhinosinusitis, atrophic rhinitis, and autoimmune processes, and it can be diagnosed, in part, by alleviation of symptoms after the placement of a moist piece of cotton in the nose. Empty nose syndrome occurs in patients who have had turbinate surgery, and overresection of the inferior turbinates is the most common cause; some authors describe inferior turbinate resection as the “classic” cause. While middle turbinate resection can also be associated with empty nose syndrome, the middle turbinates are rarely resected during rhinoplasty. Instead, they are resected during endoscopic sinus surgery.

Resection of the upper lateral cartilage can be performed to correct middle vault deviation, and the upper lateral cartilage can be affected during dorsal hump reduction. The internal nasal valve is comprised of the nasal septum and the upper lateral cartilage; it does not contribute to empty nose syndrome. Resection of septal cartilage—both for septal cartilage graft harvest and to correct underlying septal pathology—can lead to complication, including hematoma, adhesions, structural deformity, and septal perforation, among others. It is not, however, associated with empty nose syndrome. Overresection of the lower lateral cartilage can lead to structural problems, such as inadequate tip support or alar ptosis. The lower lateral cartilage is not involved in empty nose syndrome.

How well did you know this?
1
Not at all
2
3
4
5
Perfectly
7
Q

A 30-year-old woman presents to the clinic with difficulty breathing. Clinical examination shows a septal deviation with contralateral inferior turbinate hypertrophy. On application of topical nasal decongestant, there is no resolution of the inferior turbinate hypertrophy. In addition to surgical correction of the deviated septum, which of the following is the most appropriate treatment for the contralateral inferior turbinate hypertrophy?

A) Complete inferior turbinectomy
B) Inferior turbinate outfracture
C) Submucosal resection
D) No additional correction is needed

A

The correct response is Option C.

Nasal septal deviations are commonly associated with nasal obstruction. Septal deviations can lead to compensatory hypertrophy of the contralateral inferior turbinate. It is important to differentiate between mucosal and bony hypertrophy. Mucosal hypertrophy typically responds well to topical nasal decongestant. This patient had no response with topical decongestant, signifying a primarily bony hypertrophy of the inferior turbinate.

While treating airway obstruction related to nasal septal deviations, it is important to address both the nasal septal deviation and the contralateral inferior turbinate hypertrophy, because straightening of the nasal septum will lead to narrowing of the airway if the contralateral inferior turbinate hypertrophy is not addressed.

The inferior turbinate has an important role in normal nasal physiology, and complete inferior turbinectomy is not advised since it leads to dryness, bleeding, mucosal crusting, ciliary dysfunction, chronic infection, malodorous nasal drainage, or atrophic rhinitis. In patients with nasal airway obstruction secondary to bony hypertrophy of the inferior turbinate, submucosal resection is the more appropriate treatment.

In patients with primarily mucosal hypertrophy of the inferior turbinate, inferior turbinate outfracture is usually adequate.

How well did you know this?
1
Not at all
2
3
4
5
Perfectly
8
Q

A 25-year-old man is concerned about the drooping tip of his nose. On physical examination, he has a dorsal hump, poor tip support, a drooping tip, excessive columella show, and a long caudal septum. Which of the following surgical maneuvers is best to correct this patient’s nasal deformity?

A) Caudal septal tongue-in-groove technique
B) Columellar strut graft
C) Lateral crura spanning sutures
D) Medial foot plate excision
E) Transdomal suture

A

The correct response is Option A.

To correct this patient’s deformity, control of tip support, projection, and rotation is essential. A septal tongue-in-groove technique will provide more reliable control of the tip. This is similar to a tongue-in-groove septal extension graft; however, because this patient has adequate caudal septal length, the medial crura can be elevated and rotated onto the native caudal septum. If the septum has too much length, which is rare, the caudal septum can be shortened. The tradeoff in this technique is stiffness of the tip.

Columella strut is much less reliable and unnecessary because there is adequate septal length. Medial foot plate excision (removes prominent foot plates) will not correct a drooping tip. Transdomal sutures (narrow nasal tip) and lateral crura spanning sutures (straighten convex/concave lower lateral cartilages) will not correct this deformity.

How well did you know this?
1
Not at all
2
3
4
5
Perfectly
9
Q

A 36-year-old woman presents for cosmetic rhinoplasty. Examination shows a retracted columella, underprojection, and underrotation of the nasal tip, along with a prominent dorsal hump. An open rhinoplasty is planned. Which of the following grafts is most likely to preserve tip rotation with time in this patient?

A) Batten
B) Columellar strut
C) Onlay
D) Septal extension
E) Spreader

A

The correct response is Option D.

Nasal tip refinement is one of the most critical aspects of rhinoplasty surgery. Control of the position is critical for long-term results. It is influenced by surgical approach, maneuvers, and postoperative healing. Tip support is reliant on not only the crura and ligaments holding them together, but also on the grafts utilized to support the repair. The two most common grafts that provide a great deal of support for the tip are the columellar strut graft and the septal extension graft. Although both will result in some degree of loss of projection and rotation in the immediate postoperative period, both will have similar preservation of tip projection. However, the septal extension graft is noted to have better preservation of tip rotation with time in comparison with the columellar strut graft. The remaining grafts listed do not affect the projection and rotation of the nasal tip as powerfully. Septal extension graft will more effectively lengthen a shortened caudal septum more effectively than a columella strut alone.

How well did you know this?
1
Not at all
2
3
4
5
Perfectly
10
Q

Which of the following treatments for inferior turbinate hypertrophy is most likely to preserve mucosal function while maximizing symptom relief?

A) Electrocautery
B) Laser cautery
C) Partial turbinectomy
D) Submucous resection
E) Turbinate outfracture

A

The correct response is Option D.

Submucous resection of the inferior turbinate is the most likely treatment to provide symptom relief and preserve function.

The inferior turbinate warms and moisturizes the air during breathing. Surgery is indicated for patients who have nasal obstruction refractory to medical management. Surgical treatment has focused on tissue reduction.

Submucous resection removes some of the underlying bone with preservation of overlying mucosa. It provides the largest improvement in nasal airflow, and it is the only treatment that has been shown to restore mucociliary clearance and secretory production. Complications include synechiae, crusting, and bleeding.

Partial turbinectomy involves selectively trimming of the inferior turbinate. It provides long-lasting relief but it does not preserve the function of the mucosa and has high a complication rate.

Turbinate outfracture attempts to lateralize the inferior turbinate with a blunt instrument. No objective data have been found to support long-term improvement. Early recurrence occurred in more than 50% of cases.

Electrocautery and laser cautery work by creating tissue injury and subsequent fibrosis. Long-lasting results have not been reported.

How well did you know this?
1
Not at all
2
3
4
5
Perfectly
11
Q

A 27-year-old woman presents to the office because of difficulty breathing. Medical history includes closed rhinoplasty with dorsal hump removal and septoplasty 3 years ago. Physical examination shows a narrowed mid vault with an inverted V deformity. Which of the following grafts is most likely to correct the patient’s condition?

A) Alar batten
B) Dorsal onlay
C) Dorsal sidewall onlay
D) Shield
E) Spreader

A

The correct response is Option E.

This patient has collapse of the upper lateral cartilages onto the nasal septum. This is the result of the dorsal hump reduction that released the upper lateral cartilages from the septum and then unsupported allows the upper lateral cartilages to collapse inward. The inverted V deformity is the result of the upper lateral cartilages collapsing inward away from the nasal bones, exposing their underlying shape (the inverted V shape of the piriform aperture). This also results in narrowing of the lateral dorsal aesthetic lines of the nose. This narrowing of the mid-vault of the nose is seen internally as a narrowing of the internal nasal valve.

The placement of spreader grafts between the septum and upper lateral cartilages will correct the inverted V deformity, widen the dorsal aesthetic lines, and open the internal nasal valve.

Placement of an alar batten graft, dorsal onlay graft, dorsal sidewall onlay graft, and/or shield grafts will not correct the inverted V deformity, dorsal aesthetic lines, or open the internal nasal valve. Thus their placement will not correct the aesthetic or functional issue seen in this patient.

An alar batten graft can be used to support and strengthen the alae. They often will be used to strengthen an overresected lateral crura. These grafts can also correct collapse of the external nasal valve.

A dorsal onlay graft is used to augment the dorsum or hide asymmetries.

A dorsal sidewall graft is used to camouflage depressions or asymmetries of the upper lateral cartilages. These grafts, however, will not open the internal nasal valve and will not have an effect on breathing.

A shield graft is used to improve nasal tip aesthetics. It can project the tip and create tip defining points.

How well did you know this?
1
Not at all
2
3
4
5
Perfectly
12
Q

Which of the following structures is associated with the anatomical boundaries of the internal nasal valve?

A) Anterior edge of the vomer
B) Anterior middle turbinate
C) Caudal border of the upper lateral cartilage
D) Cephalic border of the lower lateral cartilage
E) Foot plate of the medial crus

A

The correct response is option C.

The internal nasal valve is the narrowest point along the upper airway. Its cross-sectional area is determined by the anatomical boundaries. These comprise the caudal border of the upper lateral cartilage superiorly, the anterior inferior turbinate posteriorly, the caudal septum medially, the nasal floor inferiorly and the pyriform margin laterally. Numerous techniques have been described to stabilize internal nasal valve narrowing by graft/stabilization of the upper lateral cartilage to prevent collapse medially

How well did you know this?
1
Not at all
2
3
4
5
Perfectly
13
Q

An otherwise healthy, 40-year-old woman has a nasal deformity requiring reconstruction. A composite auricular graft is planned. Problems in which of the following anatomic areas of the nose will most likely benefit from this kind of reconstructive method?

A) Dorsum
B) Internal nasal valve
C) Sidewall
D) Tip
E) Vestibular lining

A

The correct response is Option E.

The anatomic area of the nose where a composite auricular graft would most likely be used is in the vestibular lining. In the event of alar retraction secondary to vestibular lining shortage, composite grafts have proven to be very effective in providing cartilaginous support in addition to lining. Alar retraction is caused by a shortage of vestibular lining. This shortage may be intrinsic, but more often it is secondary to contraction from scarring or prior surgery. If the alar retraction is caused by a shortage of vestibular lining, replacement or recruitment of nasal lining is required for adequate correction. The auricular composite graft is one method of replacing vestibular lining while also providing cartilaginous support. Following harvest of a composite graft of conchal cartilage and skin, an incision is placed within the vestibule parallel to the alar rim in the area of deficiency. Dissection is carried out to release scarring and facilitate mobility for caudal repositioning of the alar rim. The composite graft is then placed as an intervening graft within the incision and sutured in place.

The other options are usually not areas where such grafts are used.

How well did you know this?
1
Not at all
2
3
4
5
Perfectly
14
Q

A 45-year-old woman presents for rhinoplasty evaluation. The patient is dissatisfied with the downward movement of her nasal tip when she smiles. Which of the following muscles is responsible for this motion?

A) Alar nasalis
B) Depressor septi nasi
C) Levator labii superioris alaeque nasi
D) Procerus
E) Transverse nasalis

A

The correct response is Option B.

The muscles of the nose contribute to its dynamic motion. The depressor septi nasi originates from the incisive fossa of the maxilla and inserts into the nasal septum and posterior aspect of the alar nasalis. It depresses the nasal septum and causes the nasal tip to move downward on animation and narrows the columellar labial angle. Release of this muscle during rhinoplasty will reduce the nasal tip depression during animation, but can also cause upper lip ptosis.

The nasalis muscle consists of two parts: the alar nasalis and the transverse nasalis. The alar nasalis, also called the pars alaris or dilator naris, originates from the maxilla and inserts along the lateral crura. It has variable development and helps to open the external nasal valve.The transverse nasalis originates from the maxilla and traverses the nose, with a thin aponeurosis over the midline. It elongates the nose and constricts the nostrils.The levator labii superior alaeque nasi originates from the nasal bone and inserts into the nostril and upper lip, and dilates the nostrils and elevates the upper lip. It is also known as Otto’s muscle, and is the longest-named mammalian muscle.The procerus muscle depresses the glabella and assists in nostril flare.

How well did you know this?
1
Not at all
2
3
4
5
Perfectly
15
Q

A patient comes to the office because he is interested in rhinoplasty. He is generally satisfied with the shape of his nose when he is in repose, but he says that his nose becomes distorted when he laughs or talks. Which of the following muscles is the most likely cause of this finding?

A) Corrugator supercilii
B) Depressor septi nasi
C) Levator labii alaeque nasi
D) Orbicularis oris
E) Procerus

A

The correct response is Option B.

The depressor septi nasi originates on the upper lip and inserts at the base of the nose on both the septum and alae. When this muscle contracts in animation, it may pull the tip of the nose down, decrease the nasolabial angle, and elevate the upper lip. The labii alaeque nasi dilates the nostrils and lifts the upper lip. The procerus muscle lies between the eyebrows and functions to depress the medial eyebrows. Contraction creates the horizontal wrinkle at the nasion. Finally the corrugator supercilii is a pyramidal shaped muscle on the medial part of the supraorbital ridge which, when contracted, moves the eyebrows medially creating vertical wrinkles between the eyes.

How well did you know this?
1
Not at all
2
3
4
5
Perfectly
16
Q

A 17-year-old woman wants improvement of a large dorsal hump, hanging columella, and bulbous tip with vertically oriented lower lateral cartilages. The patient refuses the use of septal cartilage grafts. Which of the following surgical maneuvers will best avoid a dorsal inverted V deformity?

A) Internal silicone splint
B) Lateral crus mattress suture
C) Subdomalgraft
D) Tongue-in-groove tip support
E) Upper lateral spanning sutures

A

The correct response is Option E.

The inverted V deformity refers to the visibility of the caudal edge of the nasal bones caused by collapse of upper lateral cartilages. Dorsal reduction rhinoplasty removes the structural support provided by the connection of the dorsal septum to the paired upper lateral cartilages. The lateral cartilages have a tendency to then splay, distort, and collapse posteriorly. This can narrow the internal nasal valve and cause airway narrowing as well as aesthetic distortion of the dorsal aesthetic lines. Following takedown of a dorsal hump, upper lateral spanning sutures are used to re-establish the proper relationship of the dorsal medial edges of the upper lateral cartilages and the septum.

Use of spreader grafts will also re-establish this anatomy; however, it requires harvesting cartilage from the septum, more extensive surgery, and increased complications. In patients who require a wider angle at the internal nasal valve, the excess dorsal aspect of the upper lateral cartilages can be folded over on itself to create its own spreader graft, avoiding the need for a septal graft.

Tongue-in-groove refers to overlapping the medial crura onto the caudal septum. With an adequate caudal septum, this replaces the need for a columella graft but it will not effectively prevent inverted V deformity.

A subdomal graft is used to control the shape of the nasal tip. The cartilage removed from the dorsal hump can be used for this purpose without formal septal harvesting but the risk for a dorsal inverted V deformity does not change.

Lateral crus mattress sutures are used to straighten a concave or convex lateral crus.

Internal nasal splints are used to provide temporary postoperative support of the septum following septoplasty.

How well did you know this?
1
Not at all
2
3
4
5
Perfectly
17
Q

Which of the following rhinoplasty techniques is most likely to result in alar rim notching, retraction, or collapse?

A) Cephalic trim of the lower lateral cartilages
B) Failure to place a columellar strut graft
C) Failure to place spreader grafts
D) Inadequate transdomal sutures
E) Migration of a cartilage tip onlay graft

A

The correct response is Option A.

Of all of the options, the only one that would affect the appearance of the alar rims is the maneuver that affects the lower lateral cartilages. Weakened or overly resected lower lateral cartilages would have a tendency to result in deformities of the alar rims.

Migration of a cartilage tip onlay graft, failure to place a columellar strut graft, and inadequate transdomal sutures would affect the appearance of the nasal tip.

Failure to place spreader grafts would affect the internal nasal valve but have minimal change to the appearance of the nose.

How well did you know this?
1
Not at all
2
3
4
5
Perfectly
18
Q

Which of the following deformities in a patient with nasal airway obstruction is best treated with a spreader graft?

A) Bulbous tip
B) Dorsal nasal hump
C) External orifice laxity
D) Inferior turbinate hypertrophy
E) Internal nasal valve narrowing

A

The correct response is Option E.

All of the options can create decreased airflow on inspiration and can be improved with surgical maneuvers. The spreader graft placed between the septum and upper lateral cartilages is used to increase the internal nasal valve angle, thereby increasing inspiratory nasal air flow. Septal deviation could cause airway obstruction, but would best be treated with a septoplasty to remove septal cartilage narrowing the airway on the affected side. Significant inferior turbinate hypertrophy would be treated surgically with either fracture or resection, and external skin laxity or external nasal valve collapse would best be treated with stabilization using a cartilage graft on the lateral alar segment. Placing a spreader graft to widen the internal valve would not improve airflow in any of the other choices. Dorsal nasal hump and bulbous tip are not addressed by this maneuver.

How well did you know this?
1
Not at all
2
3
4
5
Perfectly
19
Q

A 35-year-old woman is evaluated because of persistent nasal airway obstruction after undergoing nasal airway surgery and rhinoplasty 6 months ago. Physical examination, including anterior rhinoscopy, demonstrates a midline septum and nasal dorsum with no evidence of nasal valve collapse. Which of the following nasal airway procedures is most commonly associated with empty nose syndrome?

A) Inferior turbinate electrocautery
B) Inferior turbinate resection
C) Microdebrider inferior turbinoplasty
D) Radiofrequency ablation inferior turbinoplasty
E) Submucosal resection septoplasty

A

The correct response is Option B.

The nasal airway procedure most commonly associated with empty nose syndrome is inferior turbinate resection. Empty nose syndrome is a clinical syndrome characterized by recalcitrant and paradoxical symptoms of nasal obstruction and suffocation despite a widely patent nasal airway. It is encountered as a postsurgical phenomenon associated with the loss of nasal tissues. This condition is also associated with a significant psychological burden. While the etiology of the condition remains unclear, its association with nasal surgeries, particularly with surgical manipulation of the inferior turbinate, is well-documented. Appreciation of this potential clinical syndrome may help nasal surgeons avoid secondary interventions that are potentially low-yield.

While empty nose syndrome has been reported after a great variety of nasal surgeries, it is most commonly associated with over-aggressive resection of the inferior turbinate. Patients with empty nose syndrome have a significantly smaller inferior turbinate volume compared to control groups on CT analysis. While any inferior turbinate surgery could potentially result in empty nose syndrome, it is much less commonly associated with submucosal reductions of the glandular tissue of the inferior turbinate. Microdebrider inferior turbinoplasty, radiofrequency inferior turbinoplasty, and electrocautery inferior turbinoplasty all represent different energy sources to accomplish the submucosal reductions from inferior turbinate hypertrophy.

All inferior turbinoplasties, compared with inferior turbinate resections, are less commonly associated with empty nose syndrome and maintain the bony structure and mucosal coverage of the inferior turbinate. A recent meta-analysis suggests there are no differences between the two most common techniques, microdebrider and radiofrequency, in overall efficacy for nasal obstruction. Similarly, the risk for complications, including persistent nasal obstruction (including the possibility of empty nose syndrome), between these two techniques is similar. Septoplasty is the most common nasal surgery for nasal obstruction, and the submucosal technique is the most frequently employed. This technique has the advantage of maintaining the nasal mucosa while repairing the structural causes of nasal airway obstruction.

Submucosal resection septoplasty is incorrect because there are limited reports of empty nose syndrome associated with septoplasty.

How well did you know this?
1
Not at all
2
3
4
5
Perfectly
20
Q

A 26-year-old woman is evaluated because she has difficulty breathing out of her right nostril. Physical examination shows the nasal septum is deviated to the right. The nasal dorsum is straight, and the nasal tip is slightly underprojected. A septoplasty is planned. Which of the following incisions is most appropriate for accessing the septum?

A) Intercartilaginous
B) Killian
C) Marginal
D) Rim
E) Weir

A

The correct response is Option B.

A Killian incision is made 1-2 cm posterior to the caudal edge of the septum and provides access to the septum for a septoplasty. It is the most appropriate of the choices listed. While the Killian incision does not provide access to the caudal septal angle, it preserves tip support. The transfixion incision, which obtains access to the septum by incising both sides of the membranous septum at its junction with the caudal septum, is sometimes used. Both provide access to the septum, but the transfixion incision disrupts the septal ligaments, which can deproject the nasal tip inversely. When nasal tip deprojection is desired, the transfixion incision is used, and when the nasal tip is slightly under projected, a Killian incision can be considered.

The Weir incision is made at the alar base. The marginal incision is made at the caudal aspect of the lower lateral cartilage, and the rim incision is made at the rim. Though often grouped together, these incisions are distinct. The intercartilaginous incision is made between the upper lateral and lower lateral cartilages. These incisions are not used for septoplasty.

The septum can also be accessed through a transcolumellar approach, which is an open approach.

How well did you know this?
1
Not at all
2
3
4
5
Perfectly
21
Q

A 35-year-old woman comes to the office for correction of an irregularity of the nasal dorsum following rhinoplasty performed 1 year ago. The patient wants nonsurgical treatment. Hyaluronic acid filler (0.6 mL) is injected into the upper third of the nasal dorsum. Immediately after injection, the patient reports partial loss of vision and pain in the right eye. Which of the following is the most appropriate next step?

A) Application of apraclonidine eye drops
B) Intravenous infusion of tissue plasminogen activator
C) Nasal subcutaneous injection of hyaluronidase
D) Percutaneous lateral canthotomy
E) Retrobulbar injection of hyaluronidase

A

The correct response is Option E.

This patient has symptoms of an intravascular injection of hyaluronic acid (HA) causing occlusion of the central retinal artery. This complication is extremely rare; however, when it occurs, treatment must be immediately instituted because the retinal circulation needs to be restored quickly (within 60 to 90 minutes) for possible reversal of symptoms.

The first line of treatment is to bathe the retinal circulation with hyaluronidase. This is achieved with a retrobulbar injection. Using a 25-gauge needle or cannula, enter the orbit along the orbital floor between the inferior and lateral rectus muscles. Advance the needle along the orbital for at least 1 inch beyond the orbital rim and inject 2 to 4 mL of undiluted hyaluronidase. Hyaluronidase adjacent to an occluded vessel can dissolve an HA embolus.

The likely mechanism of vascular occlusion is an intra-arterial injection of filler under pressure. In this case, the filler would have entered the dorsal nasal artery and traveled retrograde to the ophthalmic artery. Once the injection pressure is released, the filler would then flow antegrade into the central retinal artery which is the terminal branch of the ophthalmic artery.

Tissue plasminogen activator would be indicated for hematologic thrombosis or blood clot embolism, not HA embolus.

Subcutaneous injection of hyaluronidase is indicated for treatment for filler-related vascular compromise secondary to extravascular compression. The goal is to dissipate the extravascular compression of the artery. However, in patients with symptoms of vision loss, the likely diagnosis is a HA embolus and therefore the first injection should be retrobulbar.

Lateral canthotomy is indicated for decompression or a retrobulbar hemorrhage. Reassurance is not a reasonable treatment option as this is an emergency. Apraclonidine is a sympathomimetic eye drop use to stimulate Müller muscle and improve botulinum toxin type A–related ptosis.

How well did you know this?
1
Not at all
2
3
4
5
Perfectly
22
Q

A 25-year-old man who underwent septorhinoplasty 6 weeks ago is evaluated because of clear nasal discharge. He notes that the drainage worsens when he bends forward or strains, and the discharge tastes salty. Physical examination shows a small amount of watery drainage from the right nostril. Examination with a nasal speculum shows no other abnormalities. Which of the following is the most appropriate next step in management?

A) MRI of the skull base
B) Testing of fluid for beta-2 transferrin
C) Testing of fluid for glucose
D) Trial of a corticosteroid nasal spray
E) Trial of an oral antihistamine

A

The correct response is Option B.

The patient in question presents with a history that is suggestive of cerebrospinal fluid (CSF) leak following septoplasty. Patients with such a leak typically have unilateral clear nasal discharge that tastes salty or metallic. Straining, Valsalva maneuver, or leaning forward typically make the drainage worse.

Commonly, rocking or twisting forces applied during the septoplasty cause a traumatic injury to the cribriform plate, with a resultant CSF leak. The injury is more common (2:1) on the right side and is more common in men.

Beta-2 transferrin testing on the CSF fluid is very specific for the injury. This protein is only found in CSF, but the test is not available in all centers. Testing the fluid for glucose is not specific and has a high false-positive rate. MRI of the skull base is not particularly helpful for CSF leaks. High-resolution CT is preferred.

Steroid nasal spray and antihistamines are not used in the management of CSF rhinorrhea.

How well did you know this?
1
Not at all
2
3
4
5
Perfectly
23
Q

A 32-year-old woman is evaluated for rhinoplasty. In the course of evaluation, the Cottle maneuver is performed. This test is most likely performed to evaluate which of the following?

A) Collapse of the external nostrils
B) Hypertrophy of the inferior turbinate
C) Patency of the internal nasal valves
D) Presence of septal perforation
E) Septal mucosal thickening

A

The correct response is Option C.

Nasal airway obstruction is a common symptom among patients presenting for rhinoplasty. Evaluation of the nasal airway should be performed in all patients presenting for rhinoplasty. The key structures that affect nasal airflow include the external and internal nasal valves, the inferior turbinates, and the nasal septum. The patient should be examined for collapse of the external nasal valves on deep inspiration, and a Cottle maneuver should be performed to evaluate patency of the internal nasal valves. Internal nasal examination is aided with the use of a nasal speculum. Oxymetazoline nasal spray facilitates mucosal constriction if mucosal edema is present. Narrowing or collapse of the internal valves with inspiration should be noted, along with inferior turbinate hypertrophy, which typically occurs on the side opposite septal deviation.

How well did you know this?
1
Not at all
2
3
4
5
Perfectly
24
Q

When a cephalic trim is performed during primary rhinoplasty, which of the following is the minimum width of caudal lower lateral cartilage that should be left behind?

A) 4 mm
B) 6 mm
C) 8 mm
D) 10 mm
E) 12 mm

A

The correct response is Option B.

The nasal tip represents a complex nasal tripod. The paired lower lateral cartilages work synergistically to provide the main structural support for the nasal tip. Tip refinement is of course common in rhinoplasty, and a cephalic trim of excess lower lateral cartilage is a classic maneuver utilized in primary rhinoplasty. The cephalic trim acts to decrease vertical height of the lateral crura and to debulk to tip. It is crucial that enough cartilage be left behind when removing excess lower lateral cartilage from the cephalic portion. Classically, 6 mm (5 to 7 mm) of rim strip is the accepted standard of how much should be left behind at minimum to prevent stability compromise of the lower lateral cartilages. Resection in excess of this can weaken the lateral crus and cause retraction, notching, and/or external valve dysfunction.

How well did you know this?
1
Not at all
2
3
4
5
Perfectly
25
Q

A 30-year-old woman wants improvement of the bulbous tip of her nose. Open rhinoplasty and thinning of the tip is planned. Intraoperative examination shows the cause of the bulbous tip is widely convex lower lateral cartilages that are thick and relatively inflexible. The domal width is greater than 6 mm. The angle of divergence is normal. Which of the following surgical maneuvers is most likely to correct the deformity?

A) Alar rim grafting
B) Cephalic trimming of the lower lateral cartilage
C) Crural spanning sutures
D) Lateral crural strut grafting
E) Subdomal spreader grafting

A

The correct response is Option D.

A bulbous/boxy tip can be caused by a convex lower lateral cartilage with a wide domal width (less than 4 mm), widened angle of divergence (less than 30 degrees), or a combination of both. This patient’s deformity is a widely convex lower lateral cartilage. The best choice is a lateral crural strut graft. This graft is usually harvested from the septum and sutured to the undersurface of the lower lateral cartilage. These grafts are strong and can reshape a thick inflexible lower lateral cartilage. Crural spanning sutures can straighten the convexity of these cartilages if the cartilages are flexible; however, in this patient it would unlikely be successful.

Transgenu sutures are often needed to further refine the domal width; however, in very boxy tips, a lateral crural strut graft would also be needed for a better result. Cephalic trim of the lower lateral cartilage is often performed to narrow the cartilage. This maneuver would not correct the deformity. When performing the cephalic trim, it is important to leave at least 6 mm of cartilage for support. An alar rim graft is a strip of cartilage places is a subcutaneous pocket along the alar rim. This graft is placed caudal to the lower lateral cartilage. It can strengthen a buckled rim but would not straighten a stiff convex lower lateral. Subdomal spreader grafts are used to correct a pinched tip deformity.

How well did you know this?
1
Not at all
2
3
4
5
Perfectly
26
Q

A 23-year-old woman of Asian ancestry comes to the office to discuss augmentation rhinoplasty to address dorsal projection. She refuses harvest of graft material from a secondary donor site. Which of the following materials would most likely provide the desired augmentation, the least surgical risk, and the greatest longevity of result?

A) Ear cartilage
B) Homologous irradiated rib cartilage
C) Porous polyethylene implant
D) Septal cartilage
E) Stacked acellular dermal matrix

A

The correct response is Option D.

There are many options for materials to use for augmentation rhinoplasty. Most options fall into one of several categories including autologous soft tissue, cartilage, and bone; processed allografts such as acellular dermal matrix; homologous grafts, most often irradiated rib; and alloplastic materials including silicone, ePTFE (expanded polytetrafluoroethylene sheeting), and porous polyethylene. There are advantages and disadvantages of each. In this case, the patient refuses a donor site outside of the nasal surgery, which eliminates the options of ear or rib cartilage and the option of cultured autologous auricular chondrocytes, as this too requires harvest of ear cartilage prior to the rhinoplasty. Homologous irradiated rib cartilage is a good alternative to autologous cartilage grafts and requires no donor site but has a somewhat higher risk profile than autologous septal cartilage. Silicone implants, and alloplastic implants in general, are popular in Asian augmentation rhinoplasties and have the advantage of availability, affordability, and no donor site, but they carry a higher risk for postoperative complications, including extrusion, skin changes, and infection, all of which create significant issues when they require secondary rhinoplasty procedures. Acellular dermal matrix has the disadvantage of a resorption rate of 20 to 30 percent, too much to perform precise corrections. An ear cartilage graft would be from a secondary site.

How well did you know this?
1
Not at all
2
3
4
5
Perfectly
27
Q

During secondary open rhinoplasty through an existing transcolumellar incision, division of which of the following arteries is most likely to result in vascular ischemia of the nasal tip?

A) Anterior ethmoid
B) Columellar
C) Dorsal nasal
D) External nasal
E) Lateral nasal

A

The correct response is Option E.

A detailed knowledge of the nasal tip blood supply is critical for safe conduct when using a transcolumellar incision during primary or secondary rhinoplasty. There are several sources of arterial blood supply to the nasal tip. Some of these can be sacrificed without compromising the viability of the nasal tip skin. Rohrich et al. determined that the nasal tip has a dual blood supply derived from the ophthalmic and facial arteries. While contributions from the ophthalmic circulation’s anterior ethmoid, dorsal nasal, and external nasal arteries are present, the dominant supply is derived from branches of the facial artery. Its branches, the columellar artery (present in 68.2% of cadavers in one study) and the lateral nasal artery (present in 100% of cadavers), are more likely to provide the nasal tip with inflow even if the ophthalmic arterial branches are sacrificed during the dissection. Regardless of the presence of a prior transcolumellar incision, the nasal tip blood supply is secure if the lateral nasal arteries are preserved. Guidelines for assuring that the lateral nasal arteries remain uninjured include “hugging” the cartilage of the lateral crura in a subperichondrial plane, limiting dissection superiorly to the level of the alar groove, limiting alar base excision to a level below the alar grooves, and limited defatting of the subdermal plane of the tip.

How well did you know this?
1
Not at all
2
3
4
5
Perfectly
28
Q

A 35-year-old man comes to the office for a consultation because he is dissatisfied with the result of a rhinoplasty performed 2 years ago. The patient reports that his nasal openings collapse on deep inspiration and his nasal tip is deformed. Physical examination shows collapse of the external nasal valve on deep inspiration and bilateral asymmetric alar rim collapse with alar retraction of 1 to 2 mm. Revision rhinoplasty is planned. Which of the following grafts is most appropriate to correct these conditions?

A) Columella strut
B) Composite alar rim
C) Lateral crural strut
D) Spreader
E) Subdomal

A

The correct response is Option C.

The lateral crural strut graft is a strip of septal cartilage that (if available for harvest) is sutured to the underside of the lower lateral cartilage and provides increased support and position control. It is a very powerful graft that can reposition lower lateral cartilages, correct alar retraction, and correct external valve collapse.

The columella strut graft is for increased tip projection and support.

The composite alar rim graft includes both skin and cartilage. This graft is used for severe alar retraction and soft tissue loss of the alar rim.

Spreader grafts are used to increase airflow through the internal nasal valve as well as straighten a deviated dorsal septum and improve dorsal aesthetic lines.

The subdomal graft is placed under the domes of the lower cartilages and can correct asymmetry of the nasal tip and improve a pinched tip.

How well did you know this?
1
Not at all
2
3
4
5
Perfectly
29
Q

A patient is evaluated because of nasal airway obstruction that is worse on the right side. Physical examination shows the inferior nasal turbinate has significant anterior extension and mucosal thickening with bony hypertrophy. There is a posttraumatic septal deviation and a 10-degree internal nasal valve angle. In consideration of surgery to improve the nasal airflow, which of the following factors is most important in determining the need for a submucous resection of the turbinate?

A) Anterior extension of the turbinate
B) Bony hypertrophy
C) Decreased internal nasal valve angle
D) Deviated septum
E) Mucosal thickening

A

The correct response is Option B.

All of the answers listed can play a part in this patient’s nasal obstruction and poor airflow in the right nostril. Of the answers listed, the bony hypertrophy of the inferior turbinate is the finding which most suggests the need for a submucous resection. The anterior extension of the inferior turbinate certainly can play a role in decreased air flow, but this by itself does not suggest the need for submucous resection. Simple mucosal thickening of the inferior turbinate without bony hypertrophy can be addressed with outfracture. Septal deviation can be addressed with septoplasty. Decreased internal nasal valve angle can be addressed with, for instance, a spreader graft.

How well did you know this?
1
Not at all
2
3
4
5
Perfectly
30
Q

At the keystone area of the nose, which of the following most accurately describes the anatomic position of the upper lateral cartilage in relation to the nasal bones?

A) Anterior to the nasal bones
B) Caudal to the nasal bones with a 1 to 2 mm fibrous gap
C) Edge to edge with the nasal bones with no overlap
D) Posterior to the nasal bones

A

The correct response is Option D.

The keystone area of the nose is where the nasal bones overlap the upper lateral cartilages. This is usually the widest part of the nasal dorsum.

At the dorsal keystone area, the nasal bones overlap the upper lateral cartilages for a distance of 4 to 14 mm.

When reducing a dorsal hump, rasping of the bone at the keystone area uncovers the underlying cartilages, which often dictate the width of this area. Once uncovered, these structures may need to be reduced for a more aesthetic dorsal line.

In large cadaver studies, all noses demonstrate that the upper lateral cartilages are posterior to the nasal bones at the keystone area, and as such all other answer choices are incorrect.

How well did you know this?
1
Not at all
2
3
4
5
Perfectly
31
Q

A 30-year-old man comes to the office because of a frontal headache and persistent watery drainage from the right nostril 2 weeks after undergoing septorhinoplasty. Which of the following is the most appropriate next step in management?

A) Place nasal packing for 48 hours
B) Start oral antihistamines
C) Start vasoconstrictor nasal spray
D) Test nasal discharge for beta-2 transferrin
E) Reassure the patient that these symptoms are normal

A

The correct response is Option D.

Postoperative cerebrospinal fluid (CSF) leak is a rare but known complication following septoplasty. It is related to an error in surgical technique, with overly forceful manipulation of the perpendicular plate region resulting in a cribriform plate defect. The cardinal symptoms are frontal headache and a clear, watery persistent rhinorrhea. If nasal packing is present, the patient may report a metallic or salty-tasting post-nasal drip. Prompt diagnosis is required to avoid complications, particularly meningitis and pneumocephalus. CSF rhinorrhea is more common on the right side, reflecting a predominance of left-sided surgical approaches. It may present in an early manner, as in this case, or have a delayed presentation; some documented reports have a 20-year delay between septoplasty and diagnosis of CSF leak.

While imaging is beneficial, initial diagnosis of CSF leak can be made with the beta-2 transferrin or Beta-trace protein testing, which are both specific and sensitive for CSF. Both are more accurate than the traditional “halo” sign or measuring the glucose level of the fluid.

The other options listed are incorrect, as they do not diagnose or effectively treat the underlying problem.

How well did you know this?
1
Not at all
2
3
4
5
Perfectly
32
Q

Which of the following structures is an anatomical component of the internal nasal valve?

A) Anterior portion of the middle turbinate
B) Caudal edge of the upper lateral cartilage
C) Cribriform plate of the ethmoid
D) Ostium of the maxillary sinus
E) Superior border of the nasal bone

A

The correct response is Option B.

The internal nasal valve is an anatomical structure composed of several parts. Its cross-sectional dimension determines the quantity of airflow that passes through the nose while breathing both at rest and during exercise. Because it is the narrowest part of the entire airway, compromise of any of its elements correlates with symptoms of nasal obstruction. The internal nasal valve is bounded by the caudal border of the upper lateral cartilage superiorly, the nasal septum medially, the floor of the nasal vestibule inferiorly, the anterior part of the inferior turbinate posteriorly, and the bony edge of the pyriform aperture laterally. Internal nasal valve narrowing may be due to one or more of several factors, including septal deviation, turbinate hypertrophy, collapse of the upper lateral cartilage due to surgical disruption of the ligaments which support it to adjacent structures, and loss of cartilage strength due to aging. A variety of techniques have been described for restoration of the internal valve function. Most of these involve stiffening the upper lateral cartilage with cartilage graft struts, submucous resection of the nasal septum, reduction of hypertrophic turbinates, or combinations of these procedures.

How well did you know this?
1
Not at all
2
3
4
5
Perfectly
33
Q

A healthy 26-year-old woman undergoes rhinoplasty using a spreader graft. Which of the following is the most likely cause of decreased airway resistance after placement of the spreader graft?

A) Decreased angle at the external nasal valve
B) Decreased area of airway
C) Decreased radius at the internal nasal valve
D) Increased angle at the external nasal valve
E) Increased radius at the internal nasal valve

A

The correct response is Option E.

A spreader graft is placed between the septum and the upper lateral cartilages. Poiseuille law states that resistance = (viscosity × length)/radius4. About half of nasal airway resistance occurs at the internal nasal valve. The internal nasal valve, formed at the junction of the septum (medially), the nasal floor (inferiorly), the inferior turbinate (laterally), and the caudal border of the upper lateral cartilages (superiorly), accounts for a significant amount of airway resistance. Maneuvers that increase the radius at the internal nasal valve will decrease resistance exponentially.

How well did you know this?
1
Not at all
2
3
4
5
Perfectly
34
Q

A 35-year-old woman is dissatisfied with the appearance of her nose. Physical examination shows parenthesis tip deformity with vertically oriented lower lateral cartilages. Which of the following grafts is most appropriate for correction of this deformity?

A) Caudal septal extension
B) Crural turnover
C) Lateral crural strut
D) Spreader
E) Spring

A

The correct response is Option C.

This patient presents with a parenthesis deformity with vertically oriented lower lateral cartilages. To correct this deformity, the lower lateral cartilages must be rotated inferiorly. A lateral crural strut graft is a strip of cartilage 3 to 4 mm in width sutured to the deep surface of the lateral crura and then either buried or sutured to the soft tissue of the pyriform aperture. In this way, the native lower lateral cartilage can be rotated inferiorly and held in place.

A caudal septal extension graft is sutured to the caudal septum and is used to control nasal tip projection as well as lengthen an overly shortened nose.

A crural turnover graft is created by folding the cephalic portion of the upper lateral cartilage inferiorly onto itself. It thereby strengthens itself and is used to support weakened or collapsed lower lateral cartilages.

Spreader grafts are placed at the dorsal edge of the septum to correct internal nasal valve collapse and support the upper lateral cartilages.

A spring graft widens the middle vault by spanning between both upper lateral cartilages.

How well did you know this?
1
Not at all
2
3
4
5
Perfectly
35
Q

A 22-year-old woman is evaluated for revision rhinoplasty. A closed approach is planned. The alar margin is in an appropriate position. On lateral view, excessive columellar show is noted. Which of the following incisions is most appropriate for management of this deformity?

A) Alar rim
B) Intercartilaginous
C) Intracartilaginous
D) Killian
E) Transfixion

A

The correct response is Option E.

On anterior/posterior view, the alar margin and columella have been described as ideally having the appearance of a gentle gull wing in flight. The columella represents the body of the gull and in this patient it is noted to be elongated with the wings (alar margin) appearing to be normal. On lateral view, the nostril should have an oval shape. A line drawn along the long axis should bisect it into equal halves with the alar rim being 1 to 2 mm above this line and the columella 1 to 2 mm below.

The patient described exhibits findings consistent with a hanging columella. The etiology of this problem is due to either a long caudal septum, long medial crura, or combination of the two. The transfixion incision is in the membranous septum at the border of the caudal septum. It is the only incision listed above that allows access to the caudal septum for excision and can also resect any redundant membranous septum that may develop as a result of the setback.

An intracartilaginous incision is made within the substance of the lateral crus of the lower lateral cartilage. It can be used in a closed approach to combine the incision for the access to the nose with the removal of the cartilage superior to the incision to accomplish a cephalic trim of the lower lateral cartilage.

A Killian incision is used for access to the septum. It is placed 1 to 2 cm posterior to the caudal border of the septal cartilage.

An alar rim incision is made in the vestibular skin just inside the border of the nostril. This incision can be used as an approach to create a pocket for a nonanatomical rim graft.

The intercartilaginous incision is made between the upper and lower lateral cartilages and would not improve columellar show.

How well did you know this?
1
Not at all
2
3
4
5
Perfectly
36
Q

A 23-year-old woman comes to the office for consultation regarding rhinoplasty because she is dissatisfied with her smile. Physical examination shows a drooping nasal tip, shortened upper lip, and transverse upper lip crease when the patient smiles. Which of the following muscles is the most likely cause of these findings?

A) Depressor anguli oris
B) Depressor septi nasi
C) Levator anguli oris
D) Levator labii superioris
E) Nasalis

A

The correct response is Option B.

The depressor septi nasi muscle is a small, paired muscle located on both sides of the nasal septum, originating from the medial crural footplates. Its action pulls the nasal tip downward and shortens the upper lip. The overactive muscle can cause the “smiling deformity” in the patient described. It is important to evaluate the smile of the rhinoplasty patient to determine whether the depressor septi nasi muscle needs to be addressed during the procedure in order to optimize results. The depressor septi nasi muscle can be released from the medial crura through the trans-nasal approach. If the patient has tethering of the frenulum, a depressor septi nasi muscle dissection and transposition can be performed through the trans-oral approach.

The depressor anguli oris originates from the mandible and inserts into the angle of the mouth. It depresses the corner of the mouth and is associated with frowning, and does not affect the nasal tip or upper lip.

The nasalis consists of two parts: transverse and alar. The transverse part arises from the maxilla, above and lateral to the incisive fossa; its fibers proceed upward and medial, expanding into a thin aponeurosis which is continuous on the bridge of the nose with that of the muscle of the opposite side, and with the aponeurosis of the procerus. The alar part is attached by one end to the greater alar cartilage, and by the other to the integument at the point of the nose. The transverse part compresses the nostrils, drawing them toward the septum. The alar part dilates, or flares, the nostrils. While the nasalis is involved in nasal movement, it does not cause the smile deformity described.

The levator anguli oris arises from the canine fossa, located under the infraorbital foramen. The muscle’s fibers insert at the mouth’s angle, and it intermingles with the zygomaticus, triangularis, and orbicularis oris muscles. Although the muscle is involved in and helps form the smile, it elevates the angle of the mouth at the corner and is not involved in nasal tip drooping and shortened upper lip.

The levator labii superioris is a broad, flat, quadrangle muscle, and may be considered as three parts extending between the lateral side of the nose and the zygoma in the infraorbital area: angular (medial), infraorbital (intermediate), and zygomatic (lateral) head. The medial part of the angular head inserts into the greater part of the lower lateral cartilages and nasal skin, and helps dilate the nostrils. The lateral part of the angular head, infraorbital, and zygomatic head all insert by merging with fibers of the orbicularis oris, and serve to elevate and evert the upper lip, the primary function of the muscle. Although it may contribute to shortening the upper lip upon activation, it does not cause nasal tip droop.

How well did you know this?
1
Not at all
2
3
4
5
Perfectly
37
Q

A 27-year-old woman comes to the office for evaluation of her nasal tip. She asks for rhinoplasty for improvement of her aesthetic appearance. Physical examination shows asymmetric projection of the nasal tip with a pinched nasal deformity. Which of the following cartilage grafts would best correct this deformity?

A) Columellar strut
B) Lateral crural strut
C) Septal extension
D) Spreader
E) Subdomal

A

The correct response is Option E.

Correction of aesthetic and functional deformities of the nasal tip requires an understanding of the underlying anatomy and the use of cartilage grafting. A pinched nasal tip deformity results from decreased interdomal distance or narrow domal arches of the lower lateral cartilages.

Subdomal grafts can be used to correct a pinched nasal tip deformity as well as asymmetry of the domes. The subdomal graft is bar shaped, spanning beneath both domes, controlling the horizontal and vertical orientation of the domes.

The columellar strut graft corrects an underprojecting tip by increasing tip projection. The lateral crural graft is utilized to correct alar contour deformities secondary to deformed lateral crus of the lower lateral cartilages. Septal extension grafts control projection, shape, and rotation of the nasal tip. The spreader graft will expand the internal nasal valve and middle one-third of the nose but not correct a pinched tip.

How well did you know this?
1
Not at all
2
3
4
5
Perfectly
38
Q

The intercartilaginous incision in rhinoplasty follows the caudal border of which of the following?

A) Alar lateral crus
B) Caudal septum
C) Lower lateral cartilage
D) Middle crus
E) Upper lateral cartilage

A

The correct response is Option E.

The intercartilaginous incision follows the caudal border of the upper lateral cartilage and is located between it and the cephalad border of the alar lateral crus. This incision may connect, and frequently does, with a transfixion incision at the caudal border of the septum at the septal angle.

How well did you know this?
1
Not at all
2
3
4
5
Perfectly
39
Q

A 30-year-old man comes to the office because of symptoms of nasal airway obstruction. Physical examination shows a septal C-shaped deformity without dorsal deviation; Cottle maneuver is negative, and external nasal valves are competent. Which of the following is the most appropriate surgical management?

A) Alar batten grafting
B) Columellar strut grafting
C) Septoplasty
D) Spreader grafting
E) Submucous septal resection

A

The correct response is Option E.

In the patient described, the most appropriate surgical management is submucous septal resection. The important structures that affect nasal airflow and lead to obstruction are the internal and external nasal valves, the inferior turbinates, and the nasal septum. According to the classifications of the deviated nose, the patient described exhibits caudal septal deviation, with a concave (C-shaped) deformity of the septum. This is the most likely cause of this patient’s obstruction symptoms. In the absence of internal (negative Cottle maneuver) and external nasal valve collapse, resection of the deviated septum is the maneuver most likely to improve the patient’s nasal airflow and alleviate obstruction symptoms. It is of the utmost importance to preserve 9 to 10 mm L-strut of septal cartilage to maintain structural integrity.

Alar batten grafts are placed in a pocket extending from the piriform aperture to a paramedian position in the alar sidewall. They prevent lateral nasal wall collapse and alar retraction during inspiration. They are also effective in providing strength and competency to the external nasal valves. The patient has competent external nasal valves, and thus, alar batten grafts alone will not address his nasal airway obstruction, which is caused by his deviated septum. Alar batten grafts are also used as an adjunct graft to correct a caudal septum deviation after septal resection.

Columellar strut graft is placed between the medial crura for nasal tip shaping and support. Open rhinoplasty approach may cause mild loss of tip projection caused by disruption of ligamentous support and increased skin undermining, and a columellar strut will help maintain tip support. It can be used to increase nasal tip projection effectively. It does not play a role in alleviating airway obstruction caused by septal deviation.

Septoplasty is the scoring of the quadrangle cartilage to influence its shape, in an attempt to straighten it. The cartilage will bend away from the scored surface. It is an important adjunct to septal resection to shapen and straighten a deviated septum. It is likely not powerful or predictable enough to correct a septal C-shaped deformity on its own and alleviate nasal obstruction.

Spreader grafts are usually paired, longitudinal grafts placed between the dorsal septum and the upper lateral cartilages in a submucoperichondrial pocket. They are used to restore or maintain the internal nasal valve, straighten a deviated dorsal septum, improve the dorsal aesthetic lines, and reconstruct an open roof deformity. They are often placed in addition to septal resection. In the patient described, without internal nasal valve collapse and a deviated septum, spreader grafts alone will not likely improve nasal airway obstruction.

How well did you know this?
1
Not at all
2
3
4
5
Perfectly
40
Q

A 25-year-old woman undergoes rhinoplasty to correct a bulbous tip. After a cephalic trim leaving 6 mm of the lower lateral cartilage, transdomal sutures, and infracture, the tip continues to look bulbous. Which of the following techniques is most likely to improve this persistent deformity?

A) Additional cephalic trimming
B) Columellar strut grafting
C) Lateral crural mattress suture
D) Shield grafting
E) Spreader grafting

A

The correct response is Option C.

The most likely cause for a persistent bulbous tip after traditional maneuvers is convexity of the lower lateral cartilages. Lateral crural mattress sutures are effective in improving this convexity. These sutures are placed spanning the convexity and then tightened to straighten the curvature. Another option would be an alar batten graft, which is a graft placed on the medial surface of the lower lateral cartilage. Additional cephalic trim could lead to weakening of the nasal tip support and would not correct the problem. Columellar strut grafting, shield grafting, and spreader grafting will not effectively improve a naturally convex lower lateral cartilage.

How well did you know this?
1
Not at all
2
3
4
5
Perfectly
41
Q

Which of the following is the most appropriate method for demonstrating objective, dynamic nasal cavity patency and nasal function?

A) Anterior rhinoscopy
B) Cottle maneuver
C) Nasal endoscopy
D) Rhinomanometry
E) Sound wave analysis

A

The correct response is Option D.

Subjectively, the nasal valve can be assessed using the Cottle test. Anterior rhinoscopy is an objective way to evaluate the nasal cavity; however, the examiner’s assessment of how much of the nasal cavity is obstructed or patent is subjective. Nasal endoscopy, CT scan, and MRI are described as tests capable to assess the nasal cavities, helping in the diagnosis of anatomical variations associated with nasal disorders. Objectively speaking, rhinomanometry is a dynamic way to assess nasal cavity patency and nasal function; it aims at establishing nasal resistance, which is the difficulty of passing air through the nose, through the measurement of transnasal pressure and airflow. Analysis of sound waves is a static way to assess nasal patency and geometry quantifying the areas of nostril cross section all the way to the nasopharynx and nasal cavity volume between the two cross-sectional areas chosen.

How well did you know this?
1
Not at all
2
3
4
5
Perfectly
42
Q

A 32-year-old Korean man comes to the office for evaluation of a wide nose with decreased projection. He desires rhinoplasty for an improved aesthetic appearance. Compared with Caucasian nasal anatomy, which of the following is most likely in this patient?

A) Height of the lower lateral cartilage is shorter
B) Height of the upper lateral cartilage is longer
C) Length of the septal cartilage is longer
D) Length of the upper lateral cartilage is shorter
E) Overlapping length of the upper lateral cartilage and the nasal bone is shorter

A

The correct response is Option A.

The cartilaginous structures of Asian noses are substantially different from those of Caucasian noses in terms of the shape, size, thickness, and relationship to other structures. The lengths of the upper and lower lateral cartilage of Asian noses are similar to those of Caucasian noses. However, the heights of the upper and lower lateral cartilage of Asian noses are shorter than those of Caucasian noses. Therefore, rhinoplasty with cephalic resection of the lower lateral cartilage in Asian noses should be approached with caution to prevent overresection. The overlapping length of upper lateral cartilage and nasal bone is similar in both Asians and Caucasians.

How well did you know this?
1
Not at all
2
3
4
5
Perfectly
43
Q

A 35-year-old man comes to the office 4 weeks after undergoing open rhinoplasty and submucous resection of a deviated septum. He reports nasal crusting, bleeding, and a whistling sound from his nose. Which of the following is the most likely diagnosis?

A) Exposed conchal bone
B) Internal nasal valve collapse
C) “L” strut fracture
D) Septal perforation
E) Submucous hematoma

A

The correct response is Option D.

Symptoms of septal perforations include crusting along the septal defect, bleeding, and whistling. The whistling sound is due to the altered airflow pattern. Perforations can be caused by trauma, cocaine snorting, and infectious or inflammatory causes. In this patient, surgical trauma is the most likely cause. Treatments for symptomatic septal perforations include flaps and grafts. Asymptomatic perforations do not require treatment.

The symptoms of internal nasal valve collapse, and submucous hematoma would be restricted airflow. “L” strut fracture or collapse would occur with an external deformity and not the symptoms described. Exposed conchal bone is caused by overresection of inferior turbinate mucosa. When performing a submucosal resection of the inferior turbinate, this would not occur with a submucous resection of the septum.

How well did you know this?
1
Not at all
2
3
4
5
Perfectly
44
Q

A 35-year-old woman is scheduled to undergo functional septorhinoplasty for nasal airway obstruction. In this patient, perioperative administration of corticosteroids is most likely to have which of the following effects on edema and ecchymosis?

Edema Ecchymosis

A)Decreased Decreased

B)Decreased No Change

C)Increased Increased

D)No Change Decreased

E)No Change No Change

A

The correct response is Option A.

Minimizing complications after rhinoplasty is a priority for every surgeon performing the procedure. Perioperative steroid administration has been shown to decrease postoperative edema and ecchymosis in a number of prospective randomized trials. In an effort to further elucidate the significance of the data and develop an evidence-based algorithm for steroid administration, a meta-analysis of the existing literature was performed. All articles were reviewed for relevant data, which were extracted, pooled, and compared. Seven prospective randomized trials investigating perioperative steroid use in rhinoplasty have been conducted and reported. Four of these studies had the same method of patient edema and ecchymosis assessment, and their data were compared. Based on results from the four relevant studies, perioperative steroid use significantly reduces postoperative edema and ecchymosis of the upper and lower eyelids at 1 day and 7 days postoperatively (P < .0001). Preoperative steroid administration decreases postoperative upper and lower eyelid edema at 1 day preoperatively, when compared with postoperative administration (P < .05). Extended dosing is superior to one-time dosing (P < .05). Perioperative steroid use decreases postoperative edema and ecchymosis associated with rhinoplasty. Preoperative administration is superior to postoperative, and extended dosing is superior to singular. Based on these results, evidence-based guidelines for perioperative steroid administration can be given.

How well did you know this?
1
Not at all
2
3
4
5
Perfectly
45
Q

A 29-year-old woman comes to the office 1 year postoperatively after rhinoplasty with slight irregularities and asymmetry of the nasal bridge and tip. Physical examination shows mild depression of the nasal dorsum and asymmetric alar domes. Injection of a calcium hydroxyapatite gel is planned. Which of the following combinations of injection depth and anatomical location is most appropriate in this patient to minimize complications?

A) Subcutaneous area into the nasal alae
B) Subcutaneous area into the nasal alar domes
C) Subperiosteal area into the nasal sidewall
D) Supraperichondrial area into the nasal dorsum
E) Supraperichondrial area into the nasal tip

A

The correct response is Option D.

Soft-tissue fillers are minimally invasive and offer an attractive alternative to revision rhinoplasty. A variety of fillers are available, including hyaluronic acid derivatives, calcium hydroxyapatite gel, and silicone. Because of the risk of adverse reactions, silicone injectables should be avoided. Hyaluronic acid-derived and calcium hydroxyapatite fillers are better tolerated but still may occasionally cause infection, necrosis, or thinning of the soft-tissue envelope. To minimize the risk of these complications, fillers should be placed in the sub-superficial musculoaponeurotic system plane just above the plane of the periosteum. This will lessen the chance of visibility and palpability. Also, use should be restricted to the nasal dorsum and nasal sidewalls. The nasal tip and alae should be avoided because necrosis is at a much higher risk. The use of soft-tissue fillers in the nose should be approached with caution.

46
Q

A 21-year-old woman comes to the office for consultation regarding rhinoplasty. She says she is dissatisfied with the tip of her nose because it is “too big and wide.” Which of the following is the most effective suture technique to achieve a more refined triangular tip in this patient?

A) Columellar septal
B) Interdomal
C) Lateral crural mattress
D) Medial crural
E) Transdomal

A

The correct response is Option E.

The first suture for a broad, bulbous tip is the transdomal suture. This suture will narrow the dome and narrow the convexity of the lateral crus with mild increased tip projection. The interdomal suture is used mainly if there is asymmetry in domal height or to reduce the interdomal width.

The columellar septal suture is used to establish tip strength and integrity, which might have been lost with a transfixion incision. Lateral crural mattress sutures are used to create lateral crural concavity. Medial crural sutures or medial crural septal sutures are used to increase or decrease tip projection.

47
Q

A 21-year-old man comes to the office because of difficulty breathing through the left nostril after he was struck in the nose during a soccer game 1 year ago. He had a nosebleed at the time but did not seek medical treatment. Physical examination shows a depressed left nasal sidewall and a buckle in the nasal septum. He has increased difficulty breathing through the left naris when the right naris is occluded, although the nostril appears open. The right nasal passage is widely patent. Closed rhinoplasty with septoplasty is planned. In addition to submucous resection of septal cartilage, which of the following is the most appropriate technique for correction of the nasal airway obstruction?

A) Lateral osteotomies with a right spreader graft
B) Left lateral osteotomy with a columellar strut
C) Left lateral osteotomy with a left spreader graft
D) Medial osteotomies with bilateral spreader grafts
E) TRight medial osteotomy with a left alar batten graft

A

The correct response is Option C.

Fracturing the nose with medial and lateral osteotomies is necessary to mobilize the bony nasal pyramid and correct the collapsed left nasal bone by out-fracturing it. A left spreader graft is also necessary to keep the left internal nasal valve open and prevent the left nasal bone from collapsing and recurrence of the deformity.

A spreader graft is not necessary on the right because the right nasal passage is widely patent. Placing bilateral spreader grafts would give the nasal dorsum a wide appearance and is not required.

Since the rhinoplasty was performed through a closed technique, the columella is not destabilized, which can happen during the open rhinoplasty technique. A columellar strut is not necessary.

The patient has left internal nasal valve collapse, not external nasal valve collapse. An alar batten graft is not indicated in this situation.

48
Q

A 29-year-old woman comes for evaluation because she is dissatisfied with the appearance of her nose. Physical examination shows internal nasal valve collapse. Rhinoplasty with spreader grafts and the use of septal cartilage is planned. Which of the following best represents the minimum amount of dorsal-caudal strut that must be retained to prevent collapse?

A) 2 mm
B) 5 mm
C) 10 mm
D) 15 mm
E) 20 mm

A

The correct response is Option C.

When harvesting septal cartilage as a graft, a minimum of 10 mm of a dorsal-caudal L-shaped strut should remain to prevent collapse. While some authors advocate a more conservative approach, leaving 15 mm, others are more aggressive, leaving as little as 8 mm. The generally accepted rule of thumb, however, is 10 mm.

49
Q

A 25-year-old man undergoes a submucous resection of the septum for airway obstruction. While the surgeon is scoring the remaining L-strut, the cartilage fractures along the dorsal strut. Reconstruction with which of the following grafts is the most appropriate next step in management?

A) Columella
B) Crural turnover
C) Dorsal onlay
D) Spreader
E) Spring

A

The correct response is Option D.

When performing a submucous resection for airway obstruction, leaving an intact L-strut is recommended for nasal support. When an L-strut fracture occurs, it should be repaired to avoid middle-third nasal collapse. The strut tends to rotate posteriorly, creating a saddle-nose deformity. Spreader grafts secured with sutures will act like a batten graft and secure the L-strut in place.

A columella graft is used to support the structure and position of the lower third of the nose.

A crural turnover graft is used to support weakened or deformed lower lateral cartilages.

A dorsal onlay graft is used for dorsal augmentation and would not adequately support the fracture.

A spring graft spans between both upper lateral cartilages and is used to widen the middle vault.

50
Q

Which of the following regions accounts for the most marked contribution to total nasal airflow resistance?

A) Choanae
B) Internal nasal valve
C) Keystone area
D) Middle meatus
E) Nasal alae

A

The correct response is Option B.

The septum, the caudal border of the upper lateral cartilage, the pyriform aperture, and the anterior border of the inferior turbinate define the internal nasal valve. It is the narrowest portion of the nasal airway and accounts for approximately 50% of nasal airway resistance.

The entrance to the nostril is not an area of resistance in particular; however, the inner nostril can contribute to resistance particularly in the secondary rhinoplasty patient or a patient with weak lower lateral cartilages. This area is called the external nasal valve and is bounded by the caudal edge of the lateral crus of the lower lateral cartilage, the soft-tissue alae, the membranous septum, and the nostril sill.

The majority of airflow in the nose is through the middle meatus. It exits through the choanae posteriorly into the nasopharynx.

The choanae can be a source of resistance in the case of congenital choanal atresia where this region is blocked by bone or soft tissue. This would typically present shortly after birth.

The keystone area is the junction of the bony and cartilaginous septum with the bony dorsum. It is a structural landmark and does not describe a region of airflow.

51
Q

A 28-year-old man who is an aspiring actor comes to the office for consultation regarding rhinoplasty. He says he feels that his nose is preventing him from being a successful actor. Examination shows a 1-mm dorsal hump and a 0.5-mm supratip depression. No abnormalities of nasal width and tip shape are noted, and nasal symmetry is acceptable. Examination of the internal airway is within the normal ranges. Which of the following is the most appropriate management?

A) External rhinoplasty with rasping of the nasal hump, osteotomy, and infracture
B) Injection of hyaluronic acid gel fillers
C) Internal rhinoplasty with hump reduction
D) Referral to psychiatric consultation
E) Tip rhinoplasty only

A

The correct response is Option D.

Body dysmorphic disorder (BDD) is a preoccupation with an imagined defect in one’s appearance, or, if a slight physical anomaly is present, the person’s concern is marked excessive. According to the diagnostic criteria in the DSM-IV, the preoccupation should last for at least one hour per day, and have clinically significant impairment in social or occupational functioning, as in this clinical case.

Approximately 5% of patients seeking aesthetic surgery have BDD. The most common preoccupation in BDD is with the nose. Between 20 and 33% of patients seeking rhinoplasty have at least some features of BDD. Previous reports suggest that rhinoplasty in these patients is associated with marked dissatisfaction and an increase in BDD symptoms, not an improvement. Interestingly, the commonly used mnemonic of SIMON to identify a BDD patient – “single, immature male, overly narcissistic” was disproven by the research of Picavet, et al., who found no relationship between sex or marital status and BDD. BDD patients do best with psychiatric help and are likely to have worsened quality of life if surgery is performed.

52
Q

A 28-year-old woman comes to the office for consultation regarding rhinoplasty. The only camera available for preoperative photographs is a handheld digital camera with a built-in flash to the left of the lens. To avoid shadows while taking photographs of the patient?s right-sided facial profile, which of the following is the optimal orientation of the camera?

A ) Horizontal orientation, flash from the left
B ) Horizontal orientation, flash from the right
C ) Oblique orientation, flash from above
D ) Vertical orientation, flash from above
E ) Vertical orientation, flash from below

A

The correct response is Option B.

Horizontal orientation of the camera with the flash coming from the right for a right-sided profile will cast the shadow behind the subject. In this case, it is necessary to invert the camera so that the flash, which is left of the lens, is now on the right, the same side as the nose.

With variations in camera position, the shadows can be markedly altered, thereby affecting the consistency of your images. Horizontal orientation with the flash from the left would cast the patient’s shadow in front of her profile. Oblique camera orientation should never be used in medical photography. Photographs should be taken orthogonally, either along the longitudinal axis of the patient or at right

53
Q

A 40-year-old man is referred for evaluation 1 year after undergoing rhinoplasty because he reports losing the sense of taste and smell. The patient states that this “complication of surgery” is affecting him in his daily activities. The patient’s inability to smell which of the following items most likely indicates that he is malingering?

A ) Alcohol
B ) Ammonia
C ) Cinnamon
D ) Licorice
E ) Mint

A

The correct response is Option B.

Although there are many prefabricated tests for anosmia, they all eventually rely on the fact that a patient who has olfactory disturbances can still identify irritants recognized by the trigeminal nervous system, eg, ammonia. In a recent study, malingering was found to be highly associated with self-reporting loss of smell and taste, involvement in litigation, and a report of broad negative effect on daily activities.

54
Q

A 32-year-old man with Bell palsy comes for evaluation of nasal obstruction. He says that the left side of his nose constantly feels clogged. Physical examination shows left facial paralysis and collapse of the left external valve. Which of the following muscles is most likely paralyzed?

A ) Depressor septi nasi
B ) Levator labii superioris
C ) Procerus
D ) Risorius
E ) Transverse nasalis

A

The correct response is Option B.

The muscles of the nose are crucial to the dynamic function of the nasal valve and airway. Patients with facial paralysis may often have symptoms of nasal airway dysfunction. The muscles of the nose are innervated by cranial nerve VII; therefore, nasal airway obstruction is noted on the ipsilateral side of the paralysis.

The levator labii superioris muscle dilates the nares. Paralysis of the muscle allows for collapse of the external valve resulting in airway obstruction.

Paralysis of these muscles would not result in collapse of the external nasal valve. The depressor septi nasi muscle depresses the nasal tip. The procerus muscle moves the eyebrows

55
Q

A 30-year-old woman comes to the office because of difficulty breathing. She requests rhinoplasty. Physical examination shows a moderate-sized dorsal hump. Nasal examination shows normal mucosa, septum, and turbinates, and an angle of less than 10 degrees between the septum and the upper lateral cartilages. Which of the following surgical maneuvers for functional airway improvement is most appropriate in this patient?

A ) Avoidance of nasal bone infracture
B ) Dorsal onlay graft
C ) Septoplasty
D ) Spreader grafts
E ) Turbinate outfracture

A

The correct response is Option D.

This patient has a narrow internal nasal valve at less than the normal 10- to 15-degree angle. This likely represents the site of the airway obstruction.

During rhinoplasty, a spreader graft may be used to open this area and give symptomatic relief to the patient’s functional issue.

Avoidance of nasal bone infracture would not correct the internal nasal valve issue. A dorsal onlay graft is used for cosmetic enhancement of the dorsal profile and would not have a functional improvement. In this case, with a normal straight septum and turbinates, modifications of these structures would not be required.

56
Q

The angle of divergence of the nasal tip is determined using which of the following structures of the lower cartilages?

A ) Middle and lateral crura
B ) Middle and medial crura
C ) Right and left foot plates
D ) Right and left lateral crura
E ) Right and left middle crura

A

The correct response is Option E.

The angle of divergence refers to the middle crura of the lower lateral cartilages. The angle of divergence is the angle between the right middle crus and the left middle crus, running from the medial genu to the lateral genu, while looking at the nose from the anteroposterior view. The angle from the middle and medial crura refers to the angle of rotation as the tip gently bends cephalad from the columella to the tip-defining point. There is no specific name given to the angle made by the lateral crura of the lower lateral cartilages. The septum and the upper lateral cartilage form the angle of the internal valve and relate to issues of occlusion of the airway. The middle and lateral crura form the lateral genu. The ideal angle of divergence is approximately 30 to 60 degrees. A more obtuse angle produces a long intercrural distance and a more “boxy” tip. A very acute angle of divergence creates a shorter intercrural distance and a narrow lobule. Optimally, the angle of rotation is approximately 60 degrees. A more obtuse angle often results in a lower nostril-lobule ratio and a more “square” tip. A shorter or absent middle crus will cause the tip to appear stubbed with inadequate projection.

57
Q

A 27-year-old man with a deviated septum and inferior turbinate hypertrophy undergoes septoplasty, inferior turbinate outfracture, and placement of bilateral spreader grafts. Preoperative evaluation showed a narrow middle vault and internal nasal valve. Cottle maneuver improved nasal airflow. Following septoplasty, a 6-mm dorsal strut and 10-mm caudal strut remain. Which of the following is the most likely complication in this patient postoperatively?

A ) External nasal valve collapse
B ) Open roof deformity
C ) Pollybeak deformity
D ) Rocker deformity
E ) Saddle-nose deformity

A

The correct response is Option E.

When performing a septoplasty procedure in which the septal cartilage will be resected, most authors recommend preserving a 1-cm L strut (1-cm caudal strut and 1-cm dorsal strut) to preserve its strength. In the patient described, only 6 mm is preserved as a dorsal strut. This results in weakening of the dorsum that can become subject to fracture, dislocation from the boney septum, or collapse caused by the force of soft-tissue contraction. A saddle-nose deformity is the result of a collapsed dorsum.

External nasal valve collapse generally results from maneuvers that weaken the lower lateral cartilage. An open roof deformity occurs when taking down the dorsal hump to the amount that there is separation between the sidewalls and the septum. This can be closed by either nasal bone infracture or the placement of spreader grafts. A pollybeak deformity is the result of fullness in the supratip area that pushes down and underprojects the nasal tip. More common etiologies for the pollybeak deformity are excess scar formation in the supratip region or inadequate resection of the lower dorsal septum.

A rocker deformity occurs after a medial osteotomy of the nasal bones that goes beyond the thick bone of the radix. It is the contour deformity that results when, upon medially repositioning the nasal bone, the portion distal to the radix rocks out laterally.

58
Q

A 38-year-old man is evaluated because of nasal airway obstruction. The obstruction has been present since he underwent functional septorhinoplasty 9 months ago. Acoustic rhinometry shows external nasal valve collapse. Which of the following is the most effective treatment of this patient’s condition?

A ) Alar batten grafting
B ) Butterfly grafting
C ) Flaring sutures
D ) Splay grafting
E ) Spreader grafting

A

The correct response is Option A.

The most common treatment for the repair of external nasal valve collapse is the placement of alar batten grafts. These grafts help to augment and strengthen the weakened or absent lateral crus of the lower lateral cartilage. Dysfunction of the external nasal valve is most often seen after overresection of the lateral crus of the lower lateral cartilage from a previous rhinoplasty, in an attempt at tip modification. Butterfly grafts, flaring sutures, splay grafts, and spreader grafts and flaps are used to correct internal nasal valve collapse.

59
Q

A 55-year-old woman comes to the office because she is dissatisfied with the appearance of her nose (shown), specifically the scars left by acne as a young adult. She has undergone scar revision by punch biopsy and closure as well as dermabrasion by four different physicians but has never been satisfied with the results. She spends approximately 1.5 hours per day putting makeup on her face before leaving the house. She has never married and feels embarrassed to be seen in public because she feels that everyone is staring at her nose. Which of the following most accurately represents the prevalence of this diagnosis in patients who undergo plastic surgery?

A) Less than 1%
B) 2 to 4%
C) 7 to 15%
D) 22 to 25%

A

The correct response is Option C.

The patient described has body dysmorphic disorder (BDD), which affects 7 to 15% of all plastic surgery patients. In this disorder, the patient’s degree of concern is far greater than the degree of actual deformity. This perception may involve the entire body or just one area. The patient is generally unaware that his or her concerns are excessive. BDD can be associated with other diagnoses, including depression, substance abuse, social phobia, and/or obsessive-compulsive disorder. The patient is preoccupied with his or her appearance so much that a significant amount of time is spent trying to camouflage or change the outward appearance with makeup. Most patients with BDD are single (70% never married), and up to 50% have suicidal ideation. Treatment involves referral to a psychiatrist where psychotherapy and pharmacotherapy are useful. Operating on these patients almost never leads to a satisfied patient; therefore, preoperative diagnosis is essential.

The prevalence of BDD has been shown to be significantly higher in the plastic surgery population than in the general population (1 to 3%). It does not appear to have a gender or cultural predilection.

60
Q

A 21-year-old woman comes to the office because of difficulty breathing through the right nostril and dissatisfaction with the appearance of a “bump” in her nose and a wide tip. History includes three untreated nasal fractures and intermittent seasonal allergic symptoms. Functional septorhinoplasty with spreader grafts and a reduction of the nasal tip and dorsum are performed. One week postoperatively, a fluid collection that tests positive for MRSA infection is noted along the nasal dorsum. Drainage is performed, and oral antibiotics are administered. The patient comes to the office for follow-up 4 weeks postoperatively, and she says she is dissatisfied with the appearance of her nose despite significant functional and cosmetic improvement. Which of the following is the most appropriate strategy to avoid further patient dissatisfaction?

A) Ask the patient to return weekly for the next 6 weeks
B) Explain to the patient that this is a normal postoperative course and ask her to return in 4 months
C) Offer to revise her surgery, explaining that she will have to pay the facility and anesthesia charges
D) Transfer care to another surgeon

A

The correct response is Option A.

Patient dissatisfaction following aesthetic surgery has many origins, including unrealistic patient expectations, inappropriate motivation for surgery, poor patient choice on behalf of the surgeon, and underlying psychopathology. Patients whose primary motivation for surgery is to resolve conflicts in interpersonal relationships and whose chief expectation is that others will change their attitudes and behavior toward them have the highest incidence of postoperative dissatisfaction.

Aesthetic septorhinoplasty cases comprise a very large portion of the dissatisfied patient population. The most common reasons include unsatisfactory results, visible irregularities or scars, continued breathing difficulty, asymmetry, “emotional distress,” and the cost of revision surgery. Of all the operations performed by aesthetic plastic surgeons, septorhinoplasty has the highest degree of unpredictability. This problem is aggravated greatly by unrealistic patient expectations, underlying psychiatric issues, and inappropriate patient selection.

Two fundamental principles must be considered when selecting the appropriate candidate for cosmetic surgery. First, the patient’s motivation for surgery must be determined. Second, the surgeon’s own motivation must be examined. Several groups of patients with certain characteristics should be avoided. These include patients with unrealistic or overly idealized expectations, excessively demanding patients, indecisive patients, immature patients, secretive patients, patients motivated to seek surgery by others, patients with unstable personalities, patients with body dysmorphic disorder, patients you simply do not like, and “surgiholics,” or “doctor shoppers.” The key to dealing with the dissatisfied patient postoperatively is proper communication and frequent contact with the patient. This is most important in patients who exhibit early decompensation and express dissatisfaction with the result. Successful communication requires empathy, compassion, and reflective listening to make sure the patients understand that their concerns are valid and important. The common denominator of litigation in plastic surgery is poor communication. Underlying the patient’s dissatisfaction is a breakdown in rapport between the patient and surgeon. Left unaddressed, this can develop into a vicious cycle of disappointment, anger, frustration, hostility, physician defensiveness and arrogance, further patient anger, and, ultimately, a visit to an attorney.

In the scenario described, a period of 4 months would be too long to wait before seeing the patient again. She may feel that the surgeon is avoiding her situation, which may cause further frustration and anger. After a reasonable “waiting period,” it may be prudent to offer the patient further surgery at a reduced or waived surgical fee. It is important to establish a revision fee structure prior to embarking on the initial surgery. Since most patient dissatisfaction is transitory and related to perioperative psychological changes, it would not be prudent to offer surgery at this point. For the same reason, it is too soon to transfer care to another surgeon. A referral would be appropriate after 3 to 4 months, provided that a good relationship is in place with that surgeon.

61
Q

The principal blood supply to the nasal tip is provided by which of the following arteries in a patient who undergoes open rhinoplasty via a transverse columellar incision?

A) Columellar
B) Lateral nasal
C) Posterior ethmoid
D) Sphenopalatine
E) Superior labial

A

The correct response is Option B.

The principal blood supply to the nasal tip following division of the columellar skin is the lateral nasal artery, a branch of the anterior ethmoid artery (internal carotid circulation). When rhinoplasty is conducted via stepped incision in the external approach, the columellar artery, a branch of the superior labial artery (external carotid circulation) component, may be abolished by division or cautery. The other options described supply blood to the posterior nasal septum (sphenopalatine artery), the upper lip (superior labial artery), and the upper central nasal septum (posterior ethmoid artery).

62
Q

A 45-year-old man comes to the office because of a chronic “stuffy nose” that is worse in the mornings than in the evenings. After decongestion, examination shows a slight posterior bony septal deviation, internal nasal valve angle of 12 degrees, and bilateral inferior turbinate hypertrophy. After a failed course of medical management, which of the following is the most appropriate single treatment?

A) Alar batten grafts
B) Flaring sutures
C) Inferior turbinate reduction
D) Septoplasty
E) Spreader grafts

A

The correct response is Option C.

While each of the listed procedures can help with nasal airway obstruction, bilateral inferior turbinate hypertrophy is its most common cause. The nasal valve consists of four distinct airflow-resistive components. The vestibule terminates in an airflow-resistive aperture between the septum and the caudal end of the upper lateral cartilage. Its cross-sectional area is stabilized by its cartilaginous structures and inspiratory isometric contractions of the alar dilator muscles. Its walls are devoid of erectile tissues that might otherwise affect its cross-sectional area and airflow resistance. By contrast, the bony entrance to the cavum is occupied by erectile tissues of both lateral (turbinates) and septal nasal walls that modulate the cross-sectional area of airway and airflow resistance. The body of the cavum offers little resistance to airflow. Valve constriction induces “orifice flow” of inspiratory air as it enters the body of the cavum, disrupting laminar characteristics and enhancing exchanges with nasal mucosa of heat, water, and contaminants. Acoustic rhinometric and rhinomanometric measurements show that it is seldom necessary to extend septal and/or turbinate surgery far beyond the piriform aperture in the treatment of nasal obstruction. The bony septal deviation is small and posterior, so septoplasty would not be the most helpful. Spreader grafts and flaring sutures help with internal nasal valve collapse, but are not the most beneficial in this instance. Alar batten grafts are used to correct external nasal valve collapse, which the patient described does not have. The normal internal valve angle is 10 to 15 degrees.

Rhinitis is the common cause of nasal obstruction. Medical treatment should be instituted for turbinate hypertrophy before committing to surgery. Rhinitis has many pathogeneses. The most common type, infectious rhinitis, is nearly always caused by a virus (rhinovirus, or the common cold). Oral or topical decongestants are helpful during the acute phase. Topical decongestants are safe, if used on a short-term basis (3 days) to avoid rhinitis medicamentosa.

Another common cause of nasal obstruction is allergic rhinitis, which is an antigen-antibody reaction mediated by immunoglobulin E. Allergic rhinitis is a seasonal affliction; symptoms occur after exposure to an airborne pollen or fungal spore. Associated symptoms include sneezing, itching, and coryza. Many medication options are available for conservative medical management, each having its own specific indications for use. The most useful classes of medications include decongestants, second-generation antihistamines, cromolyn sodium nasal spray (mast cell stabilizer), nasal topical corticosteroids, ipratropium bromide nasal spray (anticholinergic), and corticosteroid injection of the inferior turbinate. Often, these medications are used in various combinations to maximize the treatment for each patient.

63
Q

A 45-year-old woman is being evaluated because of discrete, red, facial capillaries that she would like to have removed. Which of the following lasers is most appropriate to ablate the vessels?

A ) KTP (532-595 nm)

B ) Q-switched ruby (694 nm)

C ) Nd:YAG (1064 nm)

D ) Er:YAG (2940 nm)

E ) Carbon dioxide (10,600 nm)

A

The correct response is Option A.

The 532-595 nm wavelength is the most appropriate choice, as it has the highest affinity for the vessels and can be more effective with the appropriate settings. Carbon dioxide and erbium lasers are both ablative lasers with a higher affinity for water. The 1064-nm laser can be used for hair reduction or collagen stimulation but works on the deeper layers and is more specific for darker pigmentation of vessels, such as blue. It can be used for vessel reduction in the leg; however, facial vessels are more superficial and often more red than blue in coloration. The 694nm laser would not be the first-line laser for this treatment.

64
Q

A 36-year-old woman is scheduled to undergo secondary rhinoplasty to correct an inverted-V deformity. Which of the following operative steps is most likely to correct this deformity?

A ) Infracturing of the frontal processes of the maxilla

B ) Nasal bone rasping

C ) Placement of a spreader graft

D ) Resection of the caudal septum

E ) Weir excisions

A

The correct response is Option C.

Although the inverted-V deformity is often attributed to avulsion of the upper lateral cartilages, it can also be caused by excessive removal of the transverse portion of the upper lateral cartilage during dorsal septal resection. When the transverse portion of the upper lateral cartilage is over-resected, collapse of the nasal sidewalls occurs with retraction of the upper lateral cartilage and exposure of the shape of the nasal bones in the keystone area. In the scenario described, spreader grafts fashioned from septal cartilage can be used to correct this deformity. Spreader grafts can also restore an open roof deformity after aggressive hump reduction and recreate the dorsal aesthetic lines while simultaneously maintaining patency of the internal valve. These versatile grafts can be placed unilaterally or bilaterally, visible or invisible, depending on the deformity and the desired result.

Nasal hump deformity is usually corrected with nasal bone rasping. Weir excisions are resections of the alar bases used to reduce wide or flaring nostrils. Resection of the caudal septum is used to correct the hanging columella. Osteotomy of the frontal process of the maxilla helps reduce or narrow a wide nasal bridge.

65
Q

A 26-year-old man is referred for evaluation for nasal surgery. He is satisfied with the appearance of the nose, but he has difficulty breathing through it. A Cottle maneuver significantly improves the patient €™s breathing. On examination, an anterior rhinoscopy is performed using a nasal speculum. No marked anatomic abnormalities are noted. Which of the following is the most appropriate management?

A ) Placement of a columellar strut

B ) Placement of a lateral crural strut

C ) Placement of a spreader graft

D ) Septoplasty

E ) Submucous resection of inferior turbinate

A

The correct response is Option C.

In the patient described, the improvement in respiration with the Cottle maneuver defines the area of obstruction as the internal nasal valve. The Cottle maneuver is performed by placing lateral traction on the cheek skin. This displaces the lateral nasal wall, thus opening the internal nasal valve. A normal internal nasal valve is 10 to 15 degrees. The most appropriate management for a narrowed internal nasal valve is the placement of a spreader graft to increase this angle. An anterior rhinoscopy is the inspection of the anterior portion of the nasal cavity with or without the aid of a nasal speculum.

A lateral crural strut would be indicated in cases where the obstruction is located at the external nasal valve.

Although the junction of the septum with the upper lateral cartilage contributes to the internal nasal valve, this angle will not be affected by the performance of a septoplasty. This is because the 10-mm L-strut preserved during a septoplasty is the component of the septum involved in the internal nasal valve.

Hypertrophy of the inferior turbinate is considered by many to be the most frequent cause of nasal airway obstruction; however, it does not contribute to internal nasal valve obstruction, and thus a Cottle maneuver is not diagnostic for hypertrophied inferior turbinates. Reduction of size of turbinate after administration of oxymetazoline hydrochloride (Afrin) means that conchal bone size is not a factor in airflow, and thus a submucous resection is not indicated. Therefore, although a submucous resection of the inferior turbinate may improve respiration, it will not correct the internal nasal valve obstruction.

66
Q

A 25-year-old man undergoes primary rhinoplasty and develops a septal hematoma. Which of the following is the most likely resulting nasal deformity?

A ) Midnasal asymmetry

B ) Pollybeak

C ) Retracted columella

D ) Saddle nose

A

The correct response is Option D.

All of the options are deformities following rhinoplasty. A septal hematoma may lead to a saddle nose deformity. A relatively high septal angle combined with decreased tip projection can lead to a pollybeak deformity. Columella retraction may result from over-resection of the caudal end of the septal cartilage and/or the nasal spine along with caudal rotation of the cartilaginous septum following overmobilization and/or vertical shortening. Midnasal asymmetry can result from asymmetric narrowing of the middle third, causing medialization of the lateral nasal wall.

67
Q

A 45-year-old man is scheduled to undergo submucous resection septoplasty to correct left-sided nasal airway obstruction. Which of the following complications is most likely to occur if the surgeon uses a full-transfixion incision instead of a Killian (hemi-transfixion) incision?

A ) Bilateral alar notching

B ) Decreased tip projection

C ) External nasal valve collapse

D ) Middle nasal vault collapse

E ) Saddle nose deformity

A

The correct response is Option B.

A full-transfixion incision can lead to decreased tip projection, especially if dissected down over the anterior nasal spine. Support for the nasal tip is lost. A columellar strut can help add support.

Alar notching results from over-resection of the lower lateral cartilages.

External nasal valve collapse results from weak or narrow lower lateral cartilages and is addressed by the use of batton grafts.

Middle nasal vault collapse is prevented by the use of spreader grafts.

Saddle nose deformity is created by over-resection of the dorsal septum. At least 10 mm of dorsal septum and 10 mm of caudal septum should be preserved in a submucous resection septoplasty.

68
Q

A 35-year-old man has persistent numbness of the nasal tip three years after undergoing cosmetic rhinoplasty through an endonasal approach. Injury to which of the following nerves is the most likely cause of the numbness?

A ) Anterior ethmoidal

B ) Infraorbital

C ) Infratrochlear

D ) Supraorbital

E ) Supratrochlear

A

The correct response is Option A.

The patient described has sustained an injury to the external branch of the anterior ethmoidal nerve. This structure is particularly vulnerable to damage during endonasal rhinoplasty procedures. The nerve emanates from between the nasal bone and the lateral nasal cartilage, supplying sensation to the skin at the distal nasal dorsum and tip.

The infraorbital nerve supplies sensory enervation to the lower lateral half of the nose and columellar skin, while the infratrochlear nerve supplies the cephalic portion of the nasal sidewalls and the skin overlying the radix. The supraorbital nerve also enervates the skin of the radix, while the supratrochlear nerve supplies sensation to the forehead skin.

69
Q

An open rhinoplasty that includes Weir resection for alar flare and tip modification is planned. Which of the following will decrease the risk of columellar flap necrosis?

(A) Limited lateral dissection above the alar groove
(B) Minimal dissection of the septal cartilage
(C) Minimal superior dissection of the osseocartilaginous framework
(D) No use of epinephrine in local anesthetic
(E) Use of only bipolar cautery during dissection

A

The correct response is Option A.

Limited dissection above the alar groove will spare injury to the lateral branch of the angular artery, which is one of five vessels contributing to the blood supply for the nasal tip and columella. However, it is the one source that could contribute to tissue loss if injured.

Caudal septal dissection has no influence on columellar flap blood supply in the scenario described.

Limiting superior dissection of the nasal region and sparing injury to the external nasal artery will not significantly affect the blood supply to the columella.

Routine use of epinephrine has no impact on the survival of distal tissue such as the fingertip, nasal tip, or penis. This has been substantiated in numerous studies.

Use of bipolar cautery is not necessary in open rhinoplasty as long as soft-tissue debulking of the nasal tip area is not performed.

70
Q

A 22-year-old woman undergoes an open tip septorhinoplasty. Which of the following arteries is the terminal blood supply to the skin envelope?
(A) Angular
(B) Columellar
(C) Facial
(D) Lateral nasal
(E) Superior labial

A

The correct response is Option D.

The lateral nasal artery, which is a branch of the angular artery, supplies the vascular arcade that keeps the nasal tip viable during an open tip septorhinoplasty.

The angular artery branches from the facial artery to supply portions of the cheek and nasal sidewall. The facial artery is a branch of the external carotid artery. The superior labial artery supplies the upper lip, and the columellar artery is a branch of the superior labial artery, which supplies the columella. It is divided during open tip rhinoplasty, and the skin must survive on the lateral nasal artery circulation.

71
Q

A 24-year-old man comes to the office because he has had difficulty breathing through the nose, particularly on forced inspiration, for the past six months. He underwent cosmetic rhinoplasty one year ago. He does not have seasonal allergies or sinus problems. Physical examination shows a small dorsal hump; the nose is otherwise aesthetically pleasing and well proportioned. Intranasal speculum examination shows the septum minimally deviated anteriorly to the left. Cottle maneuver is positive on the right. Which of the following is the most appropriate management?
(A) Placement of alar graft
(B) Placement of radix graft
(C) Placement of spreader graft
(D) Reduction of the component dorsal hump
(E) Resection of the submucous septum

A

The correct response is Option C.

The patient described requires placement of a spreader graft to stent the right internal nasal valve. The Cottle maneuver confirms the presence of internal nasal valve pathology. With the patient breathing quietly through the nose, the cheek is retracted laterally to open the internal nasal valve. If this improves the patient €™s breathing, the Cottle maneuver is said to be positive and suggests collapse of the internal nasal valve on that side.

Alar grafting is not appropriate because it is indicated for overresection of the lower lateral cartilages and inspiratory collapse.

Radix grafting could be used as a means of camouflaging the dorsal hump. However, there is no functional indication for radix grafting in the patient described.

Reduction of the component dorsal hump is a stepwise technique that emphasizes the integrity of the upper lateral cartilages, preserves internal nasal valve function, and therefore minimizes the need for spreader grafts in primary rhinoplasty patients. The patient described already has internal nasal valve collapse on the right side, either as a complication of the rhinoplasty or previously undiagnosed.

Resection of the submucous septum alone is not appropriate because the patient described has a minimally deviated septum anteriorly that is not contributing to the nasal airway obstruction.

72
Q

A 35-year-old man comes to the office for consultation about a white spot on the tip of the nose. He first noticed it three months ago, and it has gradually become more prominent since then. He underwent rhinoplasty one year ago. Physical examination shows a 3-mm-diameter white area on the nasal tip between the medial crura. Gentle pressure on the surrounding skin makes the white area more prominent. Which of the following is the most likely diagnosis?
(A) Devascularization of the skin as a result of surgery
(B) Displacement of the caudal septum
(C) Fungal infection of the epidermis
(D) Growth of a squamous cell carcinoma
(E) Pressure on the skin of the nasal tip from a graft

A

The correct response is Option E.

Although all the responses are possible, pressure on the skin of the nasal tip from a graft is the most likely diagnosis, especially in thin-skinned patients, and the placement of implants in an exposed position invites late complications.

Growth of a squamous cell carcinoma is not appropriate because new lesions do not usually present as a white spot. Although devascularization might show some skin discoloration, it is unlikely to be localized to a small spot. Displacement of the caudal septum is not an appropriate response, because this usually causes problems with the columella, not the tip.

73
Q

A 30-year-old woman comes to the office for consultation regarding aesthetic improvement of the nasal tip. Examination shows a narrow angulation of the lateral genu (dome) and a 90-degree angle of divergence of the middle crura with an intercrural distance of 8 mm. Which of the following is the most appropriate description of this variation?
(A) Boxy tip
(B) Lateral crus malposition
(C) Narrow lobule
(D) Nostril-lobular disproportion
(E) Supratip deformity

A

The correct response is Option A.

The boxy tip has been described as a square perimeter of the nasal base that is caused by the position of the lower lateral cartilage. The angle of divergence of the middle crura (and their length) determines the intercrural distance. A wide angle of divergence results in the lateral positioning of the domes appearing like the corners of a box. An angle of divergence of 90 degrees would give a long intercrural distance and a boxy tip. The optimal angle of divergence is approximately 30 to 60 degrees, depending on the literature.

Lateral crus malpositioning in a more cephalad direction can provide a flattened, broadened tip lacking in projection. This can lead to a boxy tip appearance without a long intercrural distance and obtuse angle of divergence. It is consistent with a type II boxy tip deformity, which is not described in this scenario. The scenario described more closely resembles a type I boxy tip deformity.

A narrow lobule would be seen with a more acute angle of divergence.

Nostril lobular disproportion relates to the medial and middle crura lengths and the relationship of the nostril length to the lobular length (ideal = 2:1).

A supratip deformity describes the fullness in the supratip area, often found after aggressive reduction rhinoplasty.

74
Q

A 36-year-old woman comes to the office because she has persistent difficulty breathing through the nose two years after having rhinoplasty. Examination shows a deviated dorsum and an open roof deformity. Which of the following is the most appropriate method of reconstruction?
(A) Alar batten graft
(B) Columellar strut
(C) Dorsal onlay graft
(D) Lateral nasal wall graft
(E) Spreader grafts

A

The correct response is Option E.
Spreader grafts are usually paired and longitudinal, placed between the dorsal septum and the upper lateral cartilages in a submucoperichondrial pocket. Spreader grafts are used to restore or maintain the internal nasal valve, straighten a deviated dorsal septum, improve the dorsal aesthetic lines, and reconstruct an open roof deformity.
Alar batten grafts are nonanatomic grafts placed in a pocket extending from the piriform aperture to a paramedian position in the alar sidewall at the site of maximal, lateral, nasal wall collapse during inspiration.
The columellar strut helps maintain tip support and increase tip projection and also aids in shaping the columellar-lobular angle.
Dorsal sidewall onlay grafts are placed along the lateral side of the nose and are different shapes and sizes depending on the indications. They are used to combat localized lateral depressions or asymmetries of the body of the nose and especially to camouflage collapse of the upper lateral cartilages.
The lateral nasal wall graft is placed in an undermined pocket between the undersurface of the lateral crus and the vestibular skin; it is stabilized by suturing it to the crus. It is used to correct alar retraction, alar rim collapse, and concave, convex, or malpositioned lateral crura.

75
Q

A 35-year-old woman comes to the office for consultation regarding correction of the bulbous appearance of the tip of the nose. She says she would like the nasal tip to be more defined. On physical examination, the lateral crus of the lower lateral cartilage points toward the medial canthus. A cephalic trim of the lower lateral cartilages is planned. Which of the following interventions is most likely to maintain external nasal valve competence?
(A) Addition of an alar rim graft
(B) Low to low lateral osteotomies of the nasal bones
(C) Transection of the medial crura and cephalic repositioning of lateral crura
(D) Use of spreader grafts
(E) Use of transdomal sutures

A

The correct response is Option A.
When the lateral crus is malpositioned and a cephalic trim is planned, external valve incompetence can result. Adding support to the alar rim improves external valve competence. Transdomal suturing is used to treat wide domal arches. This often mandates the use of alar rim grafts to support the external nasal valve from the concavity that can occur with the sutures. Adding additional support in the form of batten grafts (below the lower lateral cartilage or onlay/overlay grafts on the lower lateral cartilage) provides increased support to the external nasal valve and supports the tip or derotates the tip as desired.
Spreader grafts open the internal nasal valve, while transdomal suturing and middle crural transection can result in a pinched tip, alar notching, and decreased tip support and often do not add support to the external nasal valve. Low to low lateral osteotomies may not support external valve competence with the removal of the Webster triangle and subsequent lateral base narrowing.

76
Q

When referring to nasal anatomy, the angle of divergence is made between which of the following structures?

(A) Lateral crura of the lower lateral cartilages

(B) Middle and lateral crura of the lower lateral cartilages

(C) Middle and medial crura of the lower lateral cartilages

(D) Middle crura of the lower lateral cartilages

(E) Septum and upper lateral cartilage

A

The correct response is Option D.

The angle of divergence refers to the middle crura of the lower lateral cartilages. The angle of divergence is the angle between the right middle crus and the left middle crus running from the medial genu to the lateral genu, while looking at the nose from the anteroposterior view. The angle from the middle and medial crura refers to the angle of rotation as the tip gently bends cephalad from the columella to the tip-defining point. There is no specific name given to the angle made by the lateral crura of the lower lateral cartilages. The septum and the upper lateral cartilage form the angle of the internal valve and relate to issues of occlusion of the airway. The middle and lateral crura form the lateral genu.

The ideal angle of divergence is approximately 30 to 60 degrees. A more obtuse angle produces a long intercrural distance and a more boxy tip. A very acute angle of divergence makes a shorter intercrural distance and a narrow lobule. Optimally, the angle of rotation is approximately 60 degrees. A more obtuse angle often results in a lower nostril-lobule ratio and a more square tip. A shorter or absent middle crus will cause the tip to appear stubbed with inadequate projection.

77
Q

A 23-year old man comes to the office for follow-up examination four months after undergoing rhinoplasty and horizontal sliding osseous genioplasty with miniplate fixation. He says he has noticed increasing show of his lower teeth since the surgery. Physical examination shows ptosis of the lower lip and a deep inferior gingivobuccal sulcus. Which of the following is the most likely cause of these findings?

(A) Excessive traction by the digastric muscle

(B) Inadequate fixation of the osteotomy segment

(C) Inadequate repair of the mentalis muscle

(D) Injury to the marginal mandibular nerve

(E) Injury to the mental nerve

A

The correct response is Option C.

The patient described has a witch €™s chin deformity as a complication of a lower buccal sulcus incision for access during an osseous genioplasty. In making this incision, the mentalis muscle is divided. Leaving a cuff of muscle superiorly is helpful because it provides a good tissue base for suturing the muscle when closing the incision. This closure of the muscle helps to resuspend the soft tissue of the chin and prevents ptosis of the lower lip and soft tissues.

Excessive traction by the digastric muscle and inadequate fixation of the osteotomy segment are not likely causes for the findings in the patient described. Miniplate fixation should provide adequate fixation to the osteomy segment. Furthermore, relapse or traction of the osteotomy segment would not account for all of the findings in the patient described.

Injury to the marginal mandibular nerve is not likely to have caused the findings in the patient described because it is not in the field of dissection for the procedure.

Injury to the mental nerve would cause numbness of the lower lip on the side of the injury and would have no effect on lip position.

78
Q

A 25 year old woman is scheduled to undergo rhinoplasty using lateral nasal osteotomy using an external perforating technique. Which of the following arteries is most susceptible to injury during this procedure?

(A) Angular

(B) Anterior ethmoidal

(C) Dorsal nasal

(D) Infraorbital

(E) Lateral nasal

A

The correct response is Option A.

An external perforating osteotomy is performed at the nasofacial junction. When performing this osteotomy, care must be taken to avoid the angular artery. This artery arises from the facial artery and makes its way toward the nose. Once on the nose proper, the angular artery becomes the lateral nasal artery. The lateral nasal artery provides blood supply to the nasal tip and is not located in the region where the osteotomy is performed.

The external nasal branch of the anterior ethmoidal artery and the dorsal nasal artery supply blood to the lateral nasal tissues and dorsum. These vessels are both located on the nose itself and are above the level of the osteotomy. Finally, the infraorbital artery and its foramen are located lateral to the osteotomy site.

79
Q

In a worm’s eye view of the Caucasian nose, which of the following is the most common ratio of the lobular portion of the nose to the columella?

(A) 3:1

(B) 2:1

(C) 1:1

(D) 1:2

(E) 1:3

A

The correct response is Option D.

The lobular portion of the nose from the worm €™s eye view should be in a 1:2 ratio with the columella and nasal apertures. The lobular portion comprises one third and the collumellar portion comprises two thirds of the total distance from the tip to the base. The nostrils should have a teardrop configuration with the diameter of the base slightly larger than the diameter of the apex. The long axis of each nostril points in a slight medial direction.

80
Q

A 32-year-old woman comes to the office because she has persistent paresthesia of the lower lip three weeks after undergoing alloplastic chin augmentation via the submental approach. Which of the following is the most appropriate management?

(A) Gentle external massage

(B) Oral administration of a corticosteroid

(C) Surgical readjustment of the prosthesis

(D) Surgical release of the mentalis muscle fibers

(E) Observation

A

The correct response is Option C.

Paresthesia of the lip after insertion of an alloplastic chin prosthesis should be resolving or decreasing by two to three weeks postoperatively. If not, the prosthesis should be removed and trimmed superiorly at the level of the mental nerve foramen, or the implant wing should be positioned lower to prevent continued impingement of the mental nerve. Leaving a prosthesis in situ for eight weeks or longer with persistent numbness of the lip may result in some permanent loss of sensation. It is best to adjust prostheses as early as possible €”or by four weeks at the latest €”for satisfactory results.

81
Q

A 45-year-old man with microgenia is evaluated before undergoing augmentation genioplasty using an alloplastic prosthesis. A lateral cephalogram shows horizontal deficiency of 5 mm. A prosthesis of which of the following thicknesses is most appropriate for correction of this patient €™s deformity?

(A) 3 mm

(B) 6 mm

(C) 9 mm

(D) 12 mm

A

The correct response is Option B.

The ratio of soft-tissue response to augmentation genioplasty is approximately 0.8 to 1.0. Therefore, a 5-mm deficiency would best be corrected with a slightly thicker, 6-mm implant.

82
Q

A 36-year-old woman comes to the office for consultation regarding surgical augmentation of the chin. Which of the following postoperative outcomes is most likely to occur in this patient if a porous polyethylene prosthesis is used instead of a solid silicone prosthesis?
(A) Capsular contracture
(B) Infection
(C) Ingrowth of tissue
(D) Migration of the prosthesis
(E) Resorption of the prosthetic material

A

The correct response is Option C.

Porous polyethylene facial prostheses are less likely to migrate after implantation than solid silicone prostheses. The pore size (100- to 250-_m diameter) of porous polyethylene prostheses used in facial augmentation procedures is sufficient to allow fibrous tissue ingrowth and, thus, relative incorporation of the prostheses. Smooth surface prostheses, such as solid silicone, inevitably become encased in a fibrous tissue capsule through the host/foreign-body response. Neither porous polyethylene nor silicone prostheses are resorbed after implantation. The lack of capsule formation in porous polyethylene implants may make their explantation more difficult as compared with solid silicone prostheses.

83
Q

A 20-year-old woman has pollybeak deformity of the nose 18 months after she underwent primary rhinoplasty for reduction of a dorsal hump. Which of the following procedures is most appropriate for correction of this patient’s deformity?
(A) Injection of a corticosteroid into the soft tissue of the supratip
(B) Rasping of the radix
(C) Weir excisions
(D) Resection of the caudal septum
(E) Placement of nasal tip grafts

A

The correct response is Option E.

The pollybeak deformity, also known as a supratip deformity, is a convexity of the region just superior to the nasal tip (the supratip) when viewed in profile. The nasal tip lacks projection relative to the dorsum. The most common causes are overprojection of the caudal nasal dorsum and inadequate preservation of tip projection. This can be corrected by increasing nasal tip projection using cartilage grafts.

Injection of corticosteroids can be used to correct a supratip deformity when given within three months of the primary rhinoplasty and when the deformity is due to formation of fibrous tissue in the deadspace between the over-resected dorsum and remaining nasal skin envelope. Injection of corticosteroids is not indicated for any type of supratip deformity more than three months after the primary rhinoplasty.

Reducing the radix by rasping will not correct a pollybeak deformity and may accentuate the deformity by causing the nasal dorsum to appear more prominent relative to both the radix (root of the nose) and supratip regions.

Weir excisions are resections of the alar bases used to reduce wide or flaring nostrils.

Resection of the caudal septum is performed to correct a hanging columella and will not affect a supratip deformity.

84
Q

A 40-year-old woman undergoes rhinoplasty for correction of boxy tip deformity. Which of the following is the primary purpose of a transdomal suture during this procedure?
(A) Decrease in tip projection
(B) Improvement of columellar projection
(C) Narrowing of the domes
(D) Rotation of the tip
(E) Strengthening of the tip

A

The correct response is Option C.

Transdomal sutures are horizontal mattress sutures placed at the dome or in the lateral crus of the lower lateral cartilage during tip rhinoplasty. The primary purpose of the transdomal suture is to narrow the domes. The secondary purpose of the transdomal suture is to narrow the convexity of the lateral crura. At times, the transdomal suture may also have a tertiary effect of slight increase in tip projection.

Regarding suture technique for tip rhinoplasty, columellar projection and tip projection are more commonly affected by the interdomal suture. The columella-septal suture rotates the tip. Both the interdomal and columella-septal suture strengthen the tip. Both interdomal and transdomal sutures may control tip symmetry.

85
Q

A 34-year-old man undergoes rhinoplasty using local anesthesia. During resection of the nasal spine, the patient tells the surgeon that he feels pain. Inadequate anesthesia of which of the following nerves is the most likely cause?
(A) Anterior ethmoid
(B) Infraorbital
(C) Internal nasal
(D) Lesser palatine
(E) Nasopalatine

A

The correct response is Option E.

In the case described, the nasopalatine nerve was not adequately anesthetized. The nasopalatine nerve branches from the pterygopalatine ganglion to innervate the inferior septum and travels through the incisive foramen to join the greater palatine nerve from the palate.

The anterior ethmoid nerve supplies sensation to the tip of the nose and the lateral nasal vault. The internal nasal nerve is a branch of the anterior ethmoid nerve supplying the anterior nasal lining.

The infraorbital nerve supplies sensation to the lateral nasal walls and ala.

The lesser palatine nerve supplies sensation to the soft palate.

86
Q

A 38-year-old man who plays ice hockey undergoes rhinoplasty for correction of a deformity caused by repetitive injury to the nose. During the procedure, spreader grafts are placed. This intervention is most appropriate to achieve which of the following?
(A) Decrease the angle of the internal valve
(B) Narrow the mid vault
(C) Recreate the dorsonasal line
(D) Rotate the nasal tip
(E) Stabilize the external valve

A

The correct response is Option C.

Spreader grafts are a very useful adjunct in rhinoplasty procedures because they can recreate the dorsonasal line. The dorsonasal line extends from the eye to the nasal tip, making its aesthetics of special concern during rhinoplasty. The graft can be placed above the septal plane to be visible or below the septal plane to be more concealed. When placed above the septal line, the graft will more aggressively define the dorsal aesthetic line. Spreader grafts can be sutured to the septum and to the upper lateral cartilages at the level of the apex of the internal valve. Spreader grafts can also reconstruct the dorsonasal roof, widen the internal nasal valve, or straighten and buttress a high dorsally deviated septum. The spreader grafts would make the internal valve angle more obtuse and open the airway; they would not narrow the mid vault but make it more augmented as well as prevent or treat the inverted V deformity.

87
Q

A 68-year-old man comes to the office for consultation regarding rhinoplasty. Examination of the nose shows drooping and elongation of the tip complex. The primary cause of these findings is age-related loss of intrinsic support of which of the following structures?
(A) Columella
(B) Lower lateral cartilage
(C) Nasalis muscle
(D) Septum
(E) Upper lateral cartilage

A

The correct response is Option B.

The most significant and distinctive changes in the patient with advancing age occur in the nasal tip. Therefore, it is usually the area that needs the most refinement. This manifests as a drooping, elongated tip complex. The mechanism is multifactorial; however, it is primarily due to loss of intrinsic lower lateral cartilage support.

Other factors that contribute to nasal tip changes are: weakening or loss of suspensory ligament support with loss of medial crural support; thickening and possible ossification of cartilages, leading to greater prominence; thickening of the overlying skin and subcutaneous tissue with concomitant increased vascularity, leading to increased bulkiness and weight of the tip; and maxillary alveolar hypoplasia with resultant divergence of the medial crural feet and columellar shortening.

88
Q

A 28-year-old man comes to the office for consultation regarding a “small” chin. Which of the following is most likely to be achieved with implantation of a silicone prosthesis in this patient’s chin?
(A) Addition of height to the lower face
(B) Correction of asymmetries of the anterior mandible
(C) Effacement of the labial mental fold
(D) Enhancement of sagittal projection to the pogonion
(E) Increase in the bigonial distance

A

The correct response is Option D.

In this patient, a silicone prosthesis positioned correctly would add sagittal projection to the pogonion. Silicone chin prostheses are very popular because of their ease of insertion and low risk of complications. However, they have significant limitations. The patient has a decreased height of the lower lip and a deep labial mental fold, which would not be appreciably helped and could be potentially harmed by the placement of the prosthesis. Modest augmentation of the sagittal projection is the key advantage to a silicone prosthesis. Silicone prostheses cannot reliably improve asymmetries. A patient with a significant vertical deficiency and deep labial mental fold would be better served with an osseous genioplasty, or a rigidly fixed porous polyethylene (Medpor) prosthesis, to vertically lengthen the chin, efface the labial mental fold, and add sagittal projection. Porous polyethylene (Medpor) prostheses can also increase the bigonial width when used to augment the mandibular ramus.

89
Q

Two months after cosmetic rhinoplasty, a patient has numbness of the nasal tip. The most likely explanation is injury to which of the following nerves?

(A) Descending branch of the infraorbital
(B) Descending branch of the lesser palatine
(C) External branch of the anterior ethmoidal
(D) Medial branch of the infratrochlear
(E) Medial branch of the nasopalatine

A

The correct response is Option C.

Knowledge of the innervation of the nose is necessary for local anesthesia in rhinoplasty and nasal reconstruction.

The anterior ethmoidal nerve enters the nose near the crista galli and has two branches. The external branch emerges between the nasal bone and the lateral nasal cartilage and supplies the skin of the nasal tip and alae. It is vulnerable during tip cartilage dissection. The internal branch of the anterior ethmoidal supplies sensation to the septum and the internal nasal walls.

The infraorbital nerve supplies sensation to the cheek, lip, lower eyelid, and the upper gingiva. The nasopalatine runs anteroinferiorly on the nasal septum in a groove in the vomer. It supplies sensation to the septum and the hard palate.

The lesser palatine innervates the uvula, tonsil, and soft palate. The infratrochlear nerve supplies the skin of the radix. All of these nerves are branches of the fifth cranial nerve.

90
Q

Which of the following treatments for inferior turbinate hypertrophy is most likely to preserve mucosal function while maximizing symptom relief?

A) Electrocautery
B) Laser cautery
C) Partial turbinectomy
D) Submucous resection
E) Turbinate outfracture

A

The correct response is Option D.

Submucous resection of the inferior turbinate is the most likely treatment to provide symptom relief and preserve function.

The inferior turbinate warms and moisturizes the air during breathing. Surgery is indicated for patients who have nasal obstruction refractory to medical management. Surgical treatment has focused on tissue reduction.

Submucous resection removes some of the underlying bone with preservation of overlying mucosa. It provides the largest improvement in nasal airflow, and it is the only treatment that has been shown to restore mucociliary clearance and secretory production. Complications include synechiae, crusting, and bleeding.

Partial turbinectomy involves selectively trimming of the inferior turbinate. It provides long-lasting relief but it does not preserve the function of the mucosa and has high a complication rate.

Turbinate outfracture attempts to lateralize the inferior turbinate with a blunt instrument. No objective data have been found to support long-term improvement. Early recurrence occurred in more than 50% of cases.

Electrocautery and laser cautery work by creating tissue injury and subsequent fibrosis. Long-lasting results have not been reported.

91
Q

A 19-year-old man has numbness of the left lower lip four weeks after undergoing transoral placement of a Silastic chin implant. Physical examination shows superior displacement of the left wing of the implant. Which of the following is the most appropriate management?

(A) Injection of a corticosteroid
(B) Massage
(C) Observation
(D) Reoperation
(E) Taping

A

The correct response is Option D.

Reoperation should be done as soon as possible. The implant should be surgically revised to remove pressure on the mental nerve caused by superior displacement of the implant. In a patient with numbness, leaving an implant in place for eight weeks or more may lead to permanent loss of sensation due to fascicular pressure and may require nerve repair.

Observation without intervention could lead to permanent injury. Taping of the chin is unlikely to be effective and may put additional pressure on the mental nerve. Massage and corticosteroid injection are not appropriate because they would not correct the underlying problem and would delay surgery, which could lead to permanent injury.

92
Q

A 25-year-old woman comes to the office for postoperative follow-up 10 days after undergoing aesthetic rhinoplasty with rasping of a dorsal hump without the performance of cartilage grafts or osteotomy. Physical examination shows dorsal prominence with erythema but no fluctuance. Which of the following interventions is the most appropriate initial management?

(A) Needle aspiration and irrigation
(B) Observation
(C) Open excision
(D) Oral administration of an antibiotic
(E) Topical administration of an antibiotic

A

The correct response is Option D.

Because this patient has periostitis of the nasal dorsum, the most appropriate initial management is oral administration of an antibiotic to treat the infection. After the erythema resolves, the dorsal prominence can be surgically excised in 8 to 12 months. Studies show that shavings retained after dorsal rasping or saw osteotomy provide a nidus for periostitis. To reduce the risk of periostitis, all debris should be evacuated from the dorsum at the conclusion of dorsal rasping or saw osteotomy.

The other interventions are not appropriate for this patient. Observation delays treatment, which could lead to worsening infection. Topical administration of an antibiotic is ineffective in treating periostitis. Needle aspiration and irrigation and open excision are appropriate interventions to allow drainage and obtain cultures in the presence of fluctuance, which this patient does not have.

93
Q

Resection of the cephalic borders of the alar cartilages and caudal septum during rhinoplasty is most likely to have which of the following effects?

(A) Decrease the alar flare
(B) Lengthen the nose
(C) Lower the columella
(D) Move the tip cephalad
(E) Shorten the nasal bones

A

The correct response is Option D.

Resection of the cephalic borders of the alar cartilages and caudal septum is frequently done by directly accessing anatomic structures during open rhinoplasty or by intracartilaginous, infracartilaginous, marginal, or transfixion incisions when an intranasal approach is used. Cephalad resection of the lateral alar crus moves the tip of the nose cephalad, decreases its fullness, and increases the definition of the projecting points of the dome. During this surgery, care should be taken to avoid weakening the support of the nostril arch by overresecting.
The other effects listed do not occur with resection of the cephalic borders of the alar cartilages and caudal septum during rhinoplasty. Alar wedge (Weir) resection is commonly used to decrease alar flare. Resection of the caudal septum usually shortens the nose by allowing the tip of the nose to move cephalad with minimal change in the nasolabial angle. This maneuver also raises the columella relative to the alar margin and makes the upper lip appear longer. The nasal bones are not affected by manipulation of the soft-tissue tip-lobule complex.

94
Q

A 21-year-old man comes to the office for consultation regarding rhinoplasty because of a large dorsal hump. Effective surgical interventions to eliminate this patient’s deformity include each of the following EXCEPT

(A) augmentation of a saddle-nose deformity
(B) augmentation of the radix with a dorsal implant
(C) rasping of the hump
(D) resection of the hump, followed by osteotomy and infracturing of the nasal bones
(E) separation of the upper lateral cartilages from the nasal septum using a transmucosal incision

A

The correct response is Option E.
The nasal dorsal hump is predominently cartilaginous (57%) rather than bony (43%). Humps are classically resected by osteotomy. If an open-roof deformity is created, infracture should be additionally performed. Some surgeons prefer rasping the hump for fine control and shaping.

The cartilaginous portion of the hump consists of the nasal septum and the upper lateral cartilages. These structures can be resected as a unit sharply. Separating the upper lateral cartilages from the septum is not necessary and can compromise the support of the nose.

Some humps are prominent due to lack of height at the radix or the supratip area. For patients with a low caudal nasofrontal junction, a dorsal implant can give the illusion of a reduced hump. Similarly, if a patient has a saddle nose, correction of this defect will make a hump less conspicuous.

95
Q

Each of the following is a general characteristic of the Asian nose EXCEPT

(A) alar flare
(B) bulbous nasal tip
(C) columellar show
(D) thick subcutaneous tissue
(E) wide flat dorsum

A

The correct response is Option C.

Columellar show is not a usual characteristic of the Asian nose. Although no two noses are alike, common anatomic characteristics among Asian patients include alar flare, a bulbous nasal tip, a short retracted columella, thick subcutaneous tissue, and wide flat nasal dorsum. The base view of the nose commonly shows a flat columella-alar triangle with hanging ala and a poorly projecting nasal tip. All of these characteristics should be considered when evaluating an Asian patient for rhinoplasty.

Asian rhinoplasty is one of the most commonly performed cosmetic procedures in the world. Such surgeries usually focus on augmentation of the nasal dorsum with an alloplastic implant. Different alloplastic materials, sizes, and shapes of implants have been used with varying degrees of success. Although silicone implants for nasal augmentation have been popular throughout Asia, such implants are not as popular in the United States. An important consideration in implant surgery is the thickness of the overlying nasal soft tissues to minimize implant extrusion.

96
Q

The photographs shown above are of a 62-year-old man who sustained a nasal septal fracture in a high-speed motor vehicle collision five days ago. Following reduction of the fracture, which of the following grafts is most appropriate for correction of the deformity?

(A) Alloplastic graft
(B) Columellar strut graft
(C) Shield graft
(D) Spreader septal graft
(E) Umbrella graft

A

The correct response is Option D.

Fractures of the nose can occur in three zones. Fractures of the upper vault of the nose involve the nasal bones, ethmoid, vomer, and cephalic septal border. Fractures of the middle vault involve the upper lateral cartilages, septum, and maxilla. In patients with fractures of the lower vault of the nose, the alar cartilages and inferior septum are affected. This patient has fractures involving both the upper and middle vaults of the nose. Following reduction of the septum and nasal bones, spreader grafts should be inserted to provide support to the septum.

Compared with autologous tissues, alloplastic grafts have higher rates of exposure and infection and are relatively contraindicated in a patient with recent trauma to the nose and multiple mucoperichondrial lacerations.

Columellar strut, shield, and umbrella grafts are used for correction of deformities in the lower vault of the nose.

97
Q

Which of the following is the most likely adverse effect of performing infracture of the nasal bones?

(A) Development of an open roof deformity
(B) Development of a supratip deformity
(C) Narrowing of the internal nasal valve
(D) Narrowing of the nasal tip
(E) Retraction of the columella

A

The correct response is Option C.

Infracture of the nasal bones is performed to narrow the nasal dorsum. However, because the upper lateral cartilages are attached to the nasal bones, this technique may lead to an excessively acute angle (less than 10 degrees) between the upper lateral cartilages and the septum. Narrowing and occlusion of the internal nasal valve can result, causing breathing difficulties.

Infracture is also a method of correcting an open roof deformity.

Supratip deformities are generally caused by inadequate resection of the dorsal spine or excessive resection of the nasal dorsum.

The nasal tip and columella are relatively unaffected by infracture.

98
Q

The internal nasal valve is comprised anatomically of the septum and which of the following structures?

(A) Caudal edge of the upper lateral cartilage and pyriform aperture
(B) Inferior turbinate and nasal floor
(C) Nasal floor and caudal edge of the upper lateral cartilage
(D) Pyriform aperture and vomer
(E) Vomer and inferior turbinate

A

The correct response is Option C.

The internasal nasal valve is a triangular shaped passage that is comprised of the septum, the nasal floor, and the caudal portion of the upper lateral cartilage. Patency of the internal nasal valve is crucial for breathing. The normal angle of the internal nasal valve is 10 to 15 degrees; if the angle is less than 10 degrees, airway occlusion at the internal nasal valve may result. Insertion of spreader grafts, which are stick-like strips of cartilage, is recommended to open the angle and improve the airway. The grafts are placed submucosally between the dorsum of the septum and the upper lateral cartilage. In contrast, infracture of the nasal bones will only narrow the angle and worsen breathing.

99
Q

A 40-year-old woman is to undergo rhinoplasty to increase projection of the nasal tip. In addition to tip grafting, which of the following techniques will provide the greatest tip projection?

(A) Resection of the lateral crura
(B) Resection of the lower lateral cartilages
(C) Resection of the nasal spine
(D) Suturing of the medial crura
(E) Transfixion incision

A

The correct response is Option D.

Projection of the nasal tip can be increased by applying a cartilage graft to the tip, suturing the medial crura, and placing a strut graft between the medial crura. In addition, the caudal margin of the septum and cephalic alar rim can be resected to rotate the nasal tip and increase its projection subtly.

Tip projection is decreased by resecting the lateral and medial crura and the nasal spine. Complete transfixion incision ultimately decreases projection because it weakens nasal support.

It should be noted that procedures to correct tip projection can lead to a broad-based appearance of the nose. Weir-type alar resection or nasal sill resection is appropriate for rectifying this problem.

100
Q

A 30-year-old man is undergoing evaluation for rhinoplasty. He has a 20 pack/year history of cigarette smoking and says that he has difficulty breathing through his nose. Evaluation of this patient’s breathing difficulties should address each of the following anatomic structures EXCEPT the

(A) dorsal nasal hump
(B) internal nasal valve
(C) nasal septum
(D) nasal vestibule
(E) turbinates

A

The correct response is Option A.

In a 30-year-old man who has breathing difficulties, the internal nasal valve, nasal septum, nasal vestibule, and turbinates should be evaluated. Obstruction can result from collapse or narrowing of the internal nasal valve that occurs as a result of scarring or excessive resection of cartilage. Deviation or perforation of the septum can cause obstruction or turbulent air flow; however, it is common to have significant septal deviation with no obstructive symptoms. The position of the nostrils and nasal vestibule may also affect breathing; ptosis of the nasal tip or an obtuse nasolabial angle can cause turbulent air flow. In addition, the middle and inferior turbinates regulate breathing. Chronic hypertrophy of the inferior turbinate is the most common cause of obstruction in a patient who has had no trauma or previous surgery. If this condition does not resolve with topical administration of decongestant agents, turbinectomy should be considered. Obstruction of the nasal passages may also be caused by vasomotor or allergic rhinitis and exposure to cigarette smoke or other environmental toxins.

The dorsal nasal hump is a bony and cartilaginous prominence that does not affect internal nasal air flow.

101
Q

In a patient with functional nasal airway obstruction secondary to nasal valve collapse, findings on a Cottle test are negative. To relieve this patient’s airway obstruction, which of the following grafts is most appropriate?

(A) Batten graft
(B) Lateral crural strut graft
(C) Septal extension graft
(D) Spreader graft

A

The correct response is Option B.

Placement of a lateral crural strut graft is most likely to relieve this patient’s airway obstruction. The airway obstruction in this patient resulted from collapse of either the external nasal valve, which is formed from the soft tissues of the columella, ala, and nasal floor, or the internal nasal valve, which is the angle formed between the upper lateral cartilage and septum. Other potential causes of airway obstruction include septal deviation, turbinate hypertrophy, and the presence of nasal polyps.

A Cottle test should be performed first to pinpoint the site of collapse. This test can be performed by placing lateral traction on the paranasal skin of the left cheek, which will distract the upper lateral cartilage away from the septum and open the angle of the internal nasal valve. Any improvement in airflow is considered a positive finding and will confirm the diagnosis of obstruction of the internal nasal valve.

However, this patient has negative findings on the Cottle test, thereby localizing the obstruction to the external nasal valve. To correct the nasal valve collapse, a lateral crural strut graft should be inserted between the vestibular lining and the lateral crus of the lower lateral cartilage.

A batten graft, which is placed across the midportion of the quadrangular cartilage of the septum, will not correct either external or internal collapse. A septal extension graft is a large cartilage graft that can be anchored to the septum to control nasal tip projection. A spreader graft is placed along the dorsal septum; it extends posteriorly to improve internal nasal valve collapse.

102
Q

A 35-year-old man is undergoing surgical correction of a hanging columella. After trimming of the caudal margin of the nasal septum, which of the following is the most appropriate procedure for correction of this deformity?

(A) Caudal resection of the medial crura of the lower lateral cartilage
(B) Placement of a cartilage graft at the nasal spine
(C) Placement of a cartilage strut in the columella
(D) Resection of excess columellar skin
(E) Transposition of an alar cartilage flap into the columella

A

The correct response is Option A.

The hanging columella deformity typically results from prominence of the caudal margin of the septum or marked convexity of the caudal margin of the medial crura of the lower lateral cartilage. Although resection of the caudal margin alone is often sufficient for correction of this deformity, additional trimming may be required if the medial crura is excessively large. Following exposure of the medial crura, the excess cartilage and a portion of the mucosal lining may be removed.

Placement of cartilage grafts at the nasal spine would only further displace the columella inferiorly. Cartilage strut grafts and alar cartilage flaps are used to correct a hidden or retracted columella. Resection of excess skin and nasal lining is reserved for patients with severe deformities and would not be the most appropriate next step in this patient.

103
Q

A 19-year-old man has nasal obstruction on the left. Physical examination shows hypertrophy of the inferior turbinate. Which of the following is the most appropriate next step in management?

(A) Cottle test
(B) Evaluation for deviated septum
(C) Cephalography
(D) Needle biopsy
(E) Secretory IgA assay

A

The correct response is Option B.

The most appropriate next step in the management of this 19-year-old man with turbinate hypertrophy is evaluation for septal deviation. Because turbinate hypertrophy is also associated with nasal allergies, it is important to determine the underlying cause of the condition in order to appropriately treat it without causing excessive drying of the nasal mucosa and hemorrhage. The coronal CT scans shown above depict hypertrophy of the left inferior turbinate in a patient who has a mild septal deviation.

The Cottle test is used to diagnose collapse of the internal nasal valve, which is found at the junction of the septum and upper lateral cartilage. The caudal end of the upper lateral cartilage typically forms a 10- to 15-degree angle with the septum; if the angle is less than 10 degrees, air exchange is likely to be obstructed. The Cottle test is performed by placing lateral traction on the paranasal skin of the left cheek, which will distract the upper lateral cartilage away from the septum and open the angle of the internal nasal valve.

Cephalograms are used to identify disproportionate areas of the craniofacial skeleton in patients being considered for orthognathic surgery. The sella-nasion-point A (SNA) and sella-nasion-point B (SNB) angles are measured with this technique. Cephalograms would not be used to measure the inferior turbinates.
Needle biopsy is indicated only in rare instances in patients with intranasal malignancies, which are uncommon. In a patient who is suspected of having a neoplasm of the head and neck, direct nasal endoscopy and CT scans of the head are recommended.

Secretory IgA assay is not used in the diagnosis of turbinate hypertrophy.

104
Q

Which of the following maneuvers performed during rhinoplasty is most likely to result in a saddle nose deformity?

(A) Alar wedge resection
(B) Comminution of nasal bones during infracture
(C) Displacement of spreader grafts
(D) Excessive resection of the upper lateral cartilages
(E) Onlay grafting of the dorsal septum

A

The correct response is Option B.

Causes of the saddle nose deformity include excessive resection of the nasal dorsum leading to a loss of dorsal support, excessive resection of the septum, fracture of the perpendicular plate of the ethmoid, or comminution of the nasal bones during infracture that results in their displacement in the piriform aperture. Because this deformity is difficult to treat, appropriate measures should be taken to prevent its occurrence. Limiting the amount of dorsal hump that is resected will preserve the periosteal attachments over the nasal bones. A minimum of 1 cm of septal cartilage should remain in the dorsum, and its attachment to the perpendicular plate should not be disrupted. The surgeon should be conservative when reducing the dorsal nose, and infracture should be performed to minimize comminution. Onlay grafting of cartilage or bone is the recommended treatment of saddle nose deformity.

Alar wedge (Weir) resection is performed for correction of flared nostrils and is not likely to affect the contour of the nasal dorsum. Excessive resection of the upper lateral cartilage typically results in collapse of the middle vault; spreader grafts are inserted for correction of this deformity.

105
Q

Which of the following anatomic structures marks the path of the primary inspiratory nasal current?

(A) Inferior turbinate
(B) Middle meatus
(C) Middle turbinate
(D) Superior meatus
(E) Superior turbinate

A

The correct response is Option B.

The middle meatus marks the path of the primary inspiratory nasal current; the main flow of air within the nasal airway occurs through this structure. Functions of the nose include respiration, humidification, modification of temperature, filtration of particulate matter, olfaction, and phonation. The physiology of respiration can be characterized according to various laws of physics. Inspiratory flow is directly proportional to the radius of the nasal airway and inversely proportionate to resistance. In addition, total airflow is equal at each end of a tube. The cross-sectional area of the nasal airway is narrowest at the level of the internal nasal valve, and obstruction of the nasal airway is common at this site.

The inspiratory current is normally described as a parabolic curve through the nasal vault. Air enters at the level of the nares and gently arcs at a superior position, above the level of the inferior turbinate.

106
Q

A 25-year-old woman has numbness of the nasal tip two years after undergoing cosmetic rhinoplasty through an endonasal approach. Which of the following is the most likely cause of the numbness?

(A) Division of the descending branches of the infratrochlear nerve
(B) Division of the supratrochlear nerve
(C) Injury to the external nasal branch of the anterior ethmoidal nerve
(D) Injury to the external nasal branch of the supraorbital nerve
(E) Retraction injury to the branches of the infraorbital nerve

A

The correct response is Option C.

This patient has sustained an injury to the external nasal branch of the anterior ethmoidal nerve, a structure that is particularly vulnerable to damage during endonasal rhinoplasty procedures. The nerve emerges from between the nasal bone and lateral nasal cartilage, supplying sensation to the skin of the distal nasal dorsum and tip.

The infraorbital nerve supplies sensory innervation to the lower lateral half of the nose and columellar skin, while the infratrochlear nerve supplies the cephalic portion of the nasal side walls and the skin overlying the radix. The supraorbital nerve also innervates the skin of the radix. The supratrochlear nerve supplies sensation to the forehead skin.

107
Q

A 37-year-old man has partial airway obstruction on inspiration and a “caved-in” alar rim after undergoing cosmetic rhinoplasty for correction of a bulbous deformity of the nasal tip. Which of the following is the most appropriate management?

(A) Placement of transdomal sutures anchored to the caudal septum
(B) Septoplasty
(C) Use of alar grafts
(D) Use of cranial bone grafts, morselized and placed subcutaneously
(E) Use of middle nasal vault spreader grafts

A

The correct response is Option C.

In this patient who has a pinched nasal tip, the most appropriate surgical technique is insertion of alar grafts. A pinched nasal tip can result from excessive resection of the lower lateral alar cartilages and may lead to airway obstruction secondary to compromise of the external nasal valve. To correct the deformity, alar cartilage can be obtained from the auricle or nasal septum and then placed between and deep to the remaining lateral crura to oppose the forces leading to nasal valve collapse.

Transdomal sutures can be used to decrease nasal projection and transdomal width but not to correct a pinched nasal tip. Septoplasty would not typically improve this patient’s deformity. Cranial bone grafts are used for correction of saddle nose deformity. Spreader grafts are stick-like strips of cartilage placed in the middle nasal vault following resection of cartilage and bone to prevent internal nasal valve collapse.

108
Q

In addition to the septal cartilage, which of the following structures should be examined carefully in the evaluation of traumatic deviation of the nasal septum?

(A) Ethmoid and maxilla
(B) Ethmoid and vomer
(C) Nasal bones and sphenoid
(D) Sphenoid and vomer
(E) Vomer and nasal bones

A

The correct response is Option B.

In addition to the septal cartilage, the ethmoid and vomer should be examined carefully in a patient who is being evaluated for traumatic deviation of the nasal septum. The osseous component of the septum, which lies posterior to the cartilaginous component, is comprised primarily of the ethmoid bone in its superior portion and the vomer bone in its inferior portion. Because nasal septal deviation can be either developmental or traumatic (associated with fracture of the nasal bones and/or ethmoid complex), any person with obstruction of nasal airflow should be evaluated for deviation after other causes have been ruled out. This deviation can result from the mechanisms of buckling, fracture, or dislocation.

In patients with confirmed septal deviation, surgery should address both the cartilaginous and bony components. Treatment of the ethmoid and vomer should involve removal of bone and not refracture and repositioning. Excision of the septal cartilage will result in the formation of an L-shaped strut along the caudal and dorsal edges. Scoring of the concave cartilage will help re-establish contour.

Although the sphenoid, nasal, frontal, maxillary, and palatine bones can come into contact with the nasal septum, their contributions are peripheral and therefore do not need to be evaluated in a patient with suspected traumatic deviation.

109
Q

In a patient who has just undergone open rhinoplasty, perfusion of the nasal tip depends primarily on which of the following arteries?

(A) Angular
(B) Columellar branch of the superior labial
(C) Dorsal nasal
(D) External branch of the anterior ethmoidal
(E) Lateral nasal

A

The correct response is Option E.

The blood supply of the nasal tip and columella is derived from the contributions of five vessels; three of these vessels are derived from the external carotid artery, while two are derived from the internal carotid artery. The facial artery arises from the external carotid artery and then divides into the angular and labial arteries. The lateral nasal artery, which is located in the subdermal plexus at a point 2 mm to 3 mm superior to the alar groove, usually arises from the angular artery. One study showed this to occur uniformly in 97% of cadavers. In contrast, the columellar branch of the superior labial artery was present in only 77% of cadavers in the study. This branch is divided during open rhinoplasty. Therefore, following division of the columellar branch, the lateral nasal artery provides perfusion to the tip of the nose.

The other collateral vessels that arise from the internal carotid artery are the dorsal nasal artery and the external nasal branch of the anterior ethmoidal artery. The dorsal nasal artery perforates the orbital septum and runs downward on the side of the nose, where it anastomoses with the lateral nasal artery. The external nasal artery is the final branch of the anterior ethmoidal artery; it emerges between the nasal bone and upper lateral cartilage and then anastomoses with the nasal tip plexus. These arteries are believed to be very minor contributors to blood flow to the nasal tip.

One anatomic study of open rhinoplasty suggested that extensive defatting of the nasal tip or extended resection of the nasal alar base above the groove should not be performed during open rhinoplasty because it would result in division of the lateral nasal arteries. A subsequent study separated the alar base excisions into minimal, standard, and extended excisions. It was then shown that only the extended base excision would result in division of the lateral nasal artery. It was also found that both limited and standard excisions of the alar base resulted in preservation of the lateral nasal artery. The study again emphasized that the lateral nasal artery was the major contributor to the nasal tip plexus.

110
Q

The above photographs are of a 34-year-old woman who is disappointed with the aesthetic result 18 months after undergoing primary rhinoplasty. She says that her lower nose looks pointy and that she has nasal obstruction with deep breathing. On examination, she has alar collapse with inspiration. ***NO PICTURE***

Which of the following is the most appropriate operative management?

(A) Cartilage grafting to increase tip support
(B) Cephalic trimming of the lower lateral cartilages to increase nasal projection
(C) Osteotomies to narrow the upper nose
(D) Weir resections to decrease alar flare
(E) Trimming of the dorsal septum to correct the polybeak deformity

A

The correct response is Option A.

This patient has a polybeak deformity caused by excessive reduction during the primary rhinoplasty procedure, combined with poor projection of the nasal tip. The obstruction with inspiration is suggestive of alar collapse; the nasal tip is pinched and lacks support. The most appropriate next step in management is cartilage grafting of the nasal tip and alae to increase support.

Although the alae appear flared because of the absence of tip support, the lower lateral cartilages have already been trimmed excessively, and their appearance would only worsen with further trimming. In the same way, the upper nose appears wide because of the tip deficiency, but is instead an appropriate width, as demonstrated by the smooth curve from the rim of the brow to the nose, and would not benefit from osteotomies. Weir resections would only accentuate the pinched nasal tip. Because this patient’s problems have resulted from excessive reduction of the dorsal septum, any further reduction would only worsen the deformity.

111
Q

A 26-year-old woman who desires cosmetic rhinoplasty is scheduled to undergo rasping of the nasal hump and reshaping and grafting of the nasal tip followed by Weir excisions. Which of the following factors, if present, will decrease the likelihood of an optimal result in this patient?

(A) Mediterranean heritage
(B) Presence of a bony rather than cartilaginous hump
(C) Smoking history of one-half pack of cigarettes daily
(D) Thickened skin with prominent sebaceous glands
(E) Use of an open technique

A

The correct response is Option D.

Careful preoperative planning is an essential part of cosmetic rhinoplasty and will vary from patient to patient, depending on each patient’s anatomy and desired result following surgery. A patient who primarily desires a smaller or defined nasal tip but has thick skin and prominent sebaceous glands will not experience the postoperative shrinkage necessary to fit the altered nasal cartilage framework; consequently, the nasal tip may still be larger than is desired by the patient. Consequently, the skin thickness should be assessed during preoperative evaluation and discussed with this patient in order to ensure that her expectations are appropriate.

Although people of different heritages have different anatomic traits and possibly different desires for aesthetic outcomes, a successful rhinoplasty can generally be accomplished with adequate planning and input from the patient, whatever his or her ethnicity.

The congenital dorsal hump is often comprised of 50% bone and 50% cartilage. Although the variation from patient to patient can be significant, either type of hump may be successfully reduced; however, different methods of reduction may be required. A small bony hump may be rasped, while larger humps may require the use of a saw or chisel.

A mild-to-moderate smoking history has not been shown to affect rhinoplasty, most likely because of the excellent blood flow within the nasal plexus.

Both open and closed rhinoplasty techniques can be successfully used in cosmetic rhinoplasty. Open rhinoplasty offers better visualization of the tip cartilage but is associated with greater postoperative edema and the potential development of an external scar. However, the type of exposure should not affect the outcome of the rhinoplasty.